

Hello,
Dr. Batman
Hello Doctor, Welcome!
Profile

Name: Batman
Email: batman@gotham.com
PSYCHIATRY
(Total Questions - 280)Q.1. A 16 year old girl was brought to Accident and Emergency with the complaint of sudden loss of consciousness. This occurred while shopping with her mother a few hours ago. The episode lasted for less than a minute followed by full recovery. There was no head injury. The mother informed the staff that her daughter had not eaten anything for the last four days because she is extremely fanatic about losing weight. She began a strict diet and exercise regimen to lose weight a few months ago. Her mother appears distressed as she has tried everything to get her daughter to eat, but to no avail. The patient herself is adamant that she is fat and that her goal is to lose weight until she is 35 kg. The patient has no past medical history of note and is not taking any regular medications. On examination, the patient was found to be sweaty and wearing a baggy dress. Weight 49 kilgrams Height 165 centimetres BMI 18 kg/m2 Capillary blood glucose 2.1 mmol/L It was proposed that the patient be admitted but when the subject was broached to the patient, she began to scream and demanded to be released. What is the best treatment option in this patient?
Correct Answer : A
Regardless of the BMI, if there are medical complications such as electrolyte disturbances, hypoglycemia, or bradycardia, you should admit the patient. Since this patient has a history of loss of consciousness, low blood glucose, lacks insight, and is refusing voluntary admission, admission under the Mental Health Act would be appropriate.
An official assessment of her mental capacity would need to be performed to look for an impairment or disturbance that leaves the patient unable to make a decision.
ANOREXIA NERVOSA :
Anorexia nervosa is the most common cause of admissions to child and adolescent psychiatric wards.
It is most commonly seen in young women in which there is a marked distortion of body image, pathological desire for thinness, and self-induced weight loss by a variety of methods.
The majority of those with eating disorders are women so in the exam, you should expect a teenage female in the stem.
Features :
• BMI <17.5 or < 85% of that expected
• Self-induced weight loss reduces food intake, vomiting, purging, and excessive exercise
• Intense fear of being obese
• Disturbance of weight perception
• Endocrine disorders that cause amenorrhoea, reduced sexual interest/impotence, raised GH levels, raised cortisol, altered TFTs, abnormal insulin secretion
• Bradycardia
• Hypotension
• Fatigue
• Muscle weakness
• Intolerance to cold.
Q.2. A 22 year old woman thinks she is overweight. She has a body mass index of 21.8 kg/m2. She often has constipation and abdominal pain. She eats uncontrollably and feels guilty resulting in a self-induced vomiting. Sometimes to compensate for her big meal, she would exercise intensively. What is the most likely diagnosis?
Correct Answer : A
Bulimia nervosa is an eating disorder characterized by repeated episodes of uncontrolled overeating (binges) followed by compensatory weight loss behaviors.
Features:
• Excessive preoccupation with body weight and shape
• Compensatory weight control mechanisms which can be:
• Self-induced vomiting
• Fasting
• Intensive exercise
• Abuse of medication such as laxatives, diuretics, thyroxine or amphetamines.
Note that a person with bulimia nervosa does not necessarily need to be thin. They sometimes maintained a BMI above 17.5 kg/m2.
Examination:
• Salivary glands (especially the parotid) may be swollen.
• Russell's sign may be present (calluses form on the back of the hand, caused by repeated abrasion against teeth during inducement of vomiting).
• There may be erosion of dental enamel due to repeated vomiting.
Q.3. A 34 year old ptient was admitted for severe depression and suicidal thoughts. He attempted to commit suicide. He has been on sertraline for the past year which he had stopped a month ago. He had been admitted in the psychiatric ward two years ago for mania. On discharge, the patient wishes to know if he can be started on a long-term medication to alleviate his symptoms. What medication will be most useful for his disorder?
Correct Answer : B
The patient is suffering from bipolar disorder. Long-term management requires mood stabilizers and therefore lithium is the best option.
Bipolar affective disorder (commonly known as manic depression) :
Classically, periods of prolonged and profound depression alternate with periods of excessively elevated and irritable mood, known as mania.
Most people who battle with the effects of the disorder would rather live a normal life, free from the unpredictability of mood swings, which most of us take for granted.
The symptoms of mania characteristically include:
• Decreased need for sleep
• Pressured speech
• Increased libido
• Reckless behaviour without regard for consequences
• Grandiosity
• More talkative than usual
These symptoms of mania would alternate with depression.
Treatment Mood stabilizers (Lithium)
Despite problems with tolerability, lithium still remains the gold standard in the treatment of bipolar affective disorder.
Q.4. A 24 year old depressed man has neglected his personal hygiene and physical health. He denies the existence of his bowels and believes that his bowels are blocked. He also believes that his limbs are missing and that no one cares about it. What kind of delusion is he suffering from?
Correct Answer : A
Nihilistic delusion is the delusional belief that the patient has died or no longer exists or that the world has ended or is no longer real. Nothing matters any longer and continued effort is pointless. It is a feature of psychotic depressive illness. Patients may believe that he/she is dead and may ask people to bury them.
Delusion of guilt involves feeling guilty or remorseful for no valid reason. An example would be someone who believes they were responsible for a war in another country or hurricane damage in another state. The object of delusion believes that they deserve to be punished for their sins.
Persecutory delusion is a delusional belief that one’s life is being interfered with in a harmful way. It refers to false beliefs or perceptions in which a person believes that they are being treated with malicious intent, hostility, or harassment despite significant evidence to suggest otherwise. This may occur in the context of being tormented, followed, or spied on.
Frégoli delusion is when a person holds a delusional belief that different people are a single person who changes appearance or is in disguise.
Q.5. A 6 year old child is brought to the clinic by his mother. She says that his teacher complains that he is easily distracted and interruptes others when it is their turn to answer questions. His mother says that he is not able to do a particular task for a long time and cannot play quietly. He is careless and often breaks things. What is the most likely diagnosis?
Correct Answer : D
Attention deficit hyperactivity disorder (ADHD) is characterized by the three core symptoms of inattention, hyperactivity, and impulsiveness.
Clinical features :
- Inattention, careless with detail, fails to sustain attention, appears not to listen, fails to finish tasks, poor self-organization, loses things, forgetful, easily distracted, and avoids tasks requiring sustained attention.
- Hyperactivity: fidgets with hands or feet, leaves seat in the class, runs/climbs about, cannot play quietly, ‘always on the go’.
- Impulsiveness: Talks excessively, blurts out answers, cannot await turn, interrupts others, intrudes on others.
Q.6. A 26 year old man strongly believes that every elderly man he meets is likely to be his father. Although they look different, he is sure that it is his father wearing a different disguise. What is the most likely kind of delusion this man is suffering from?
Correct Answer : C
Frégoli delusion is when a person holds a delusional belief that different people are a single person who changes appearance or is in disguise.
Q.7. A couple attends a marriage counselling session because of marital problems. The wife states that her husband is having affairs although she has no proof of this. The husband states that she is “insane” because she is having him followed by a private detective and she is overly preoccupied about him being unfaithful. She goes through his personal belongings almost every day to look for signs of infidelity. Her actions are putting considerable strain on their marriage. What is the most likely syndrome she is suffering from?
Correct Answer : B
Othello Syndrome is a type of delusional jealousy, marked by suspecting a faithful partner of infidelity like cheating, adultery, or having an affair. The patient may attempt to monitor his spouse or partner.
Q.8. A 48 year old man attends his GP surgery. He was started on fluoxetine 8 weeks ago for depression and is now requesting to stop his medication as he feels well and does not think he is depressed any longer. What is the most appropriate advice to give to him in regards to his treatment?
Correct Answer : A
If a patient makes a good response to antidepressant therapy they should continue on treatment for at least 6 months after remission as this reduces the risk of relapse. When stopping an SSRI the dose should be gradually reduced over 4 weeks (this is not necessary with fluoxetine as it has a long half-life).
Q.9. A 29 year old male presents to Accident & Emergency with the complaint of neck stiffness. The patient’s past medical history is significant for schizophrenia, diagnosed a few months ago. His notes mention that his psychiatrist had just started him on a new drug called clozapine due to his previous medication not working. He is on no other medication and denies the use of over the counter medications or illicit drugs. He smokes around five cigarettes a day but does not drink alcohol. Upon physical examination, the patient appears to be working hard to breath. Active and passive flexion and extension of his neck is extremely difficulty to achieve. The patient’s face also appears red and flushed. He also appears to be confused. His vital signs are as follows: Blood pressure 168/98 mmHg Heart rate 114 beats per minute Temperature 39.2 C Oxygen saturation on room air 95%. What is the most likely diagnosis?
Correct Answer : C
This patient has neuroleptic malignant syndrome (NMS). NMS describes a rare and unusual reaction to antipsychotic drugs. The cause of NMS in this case is the patient’s use of clozapine.
Symptoms of NMS usually include high fevers, confusion, agitation, muscle rigidity, sweating, and tachycardia. The onset of these symptoms usually occurs after a few weeks of starting the medication but the symptoms can occur at any time.
Q.10. A 33 year old schizophrenic says the following. “Life is unfair, I eat air, law chair, I like fairs, fairs have foot, it must be good, in adulthood, I misunderstood”. What term describes this patient’s speech?
Correct Answer : C
Clang association is an abnormality of speech where the connection between words is their sound rather than their meaning. This may occur during the manic flight of ideas.
Clang associations generally sound a bit like rhyming poetry, except that the poems don't seem to make any sense. For example, one may say “Systematic, sympathetic, quite pathetic, apologetic, paramedic, your heart is prosthetic.
Q.11. An 18 year old male washes his hands 6 times every time he uses the toilet. On his way out of the toilet, he has to switch off the light, turn it back on and turn it off once more. What is the most appropriate management?
Correct Answer : D
Obsessive-compulsive disorder (OCD) is a common, chronic condition, often associated with marked anxiety and depression, characterized by ‘obsessions’. It is characterized by recurrent obsessions or compulsions that are recognized by the individual as unreasonable. Obsessions are anxiety-provoking, intrusive thoughts, commonly concerning contamination, doubt, guilt, aggression, and sex.
Compulsions are peculiar behaviors that reduce anxiety, commonly hand-washing, organizing, checking, counting, and praying.
Management CBT is recommended by guidelines, but essentially takes a behavioral approach, including exposure and response prevention (ERP).
Q.12. A 55 year old man has a firm belief that the headlines in the newspapers are written especially for him. He believes that the authors of the newspaper articles whom he has never met are sending secret and significant messages that only he can understand. What is the most likely type of delusion that this man is suffering from?
Correct Answer : B
Delusion of control is the false belief that another person, group of people, or external force controls one's general thoughts, feelings, impulses, or behavior.
Delusion of reference is the false belief that insignificant remarks, events, or objects in one's environment have personal meaning or significance. For example, someone constantly gives him or her a special message through the newspaper.
Persecutory delusion is a delusional belief that one’s life is being interfered with in a harmful way. It refers to false beliefs or perceptions in which a person believes that they are being treated with malicious intent, hostility, or harassment despite significant evidence to suggest otherwise.
Q.13. A 38 year old woman with a history of severe depression and bipolar affective disorder presents to the Emergency Department with coarse tremors and muscular twitching. She feels nauseous and drowsy. She had vomited three times a few hours before presenting to the Emergency Department. She has brought in a bag full of the medications she would normally take with one of the medication boxes completely empty. Which is the most likely medication responsible for her symptoms?
Correct Answer : D
Lithium toxicity is a frequently asked topic. Patients would present with symptoms of tremors, nausea, and vomiting followed by drowsiness, confusion, and later a coma. In these cases where lithium toxicity is suspected, urea creatinine, and lithium levels should be obtained.
Note that sodium valproate is also another cause of tremors but it is seen commonly with therapeutic doses rather than an overdose.
One should be very careful when choosing sodium valproate as the cause of tremors over lithium in a situation where an overdose is suspected. With sodium valproate overdose, they may also present with nausea and drowsiness just like in this stem, however, a coarse tremor is rarely seen.
Q.14. A 17 year old woman has been diagnosed with anorexia nervosa. She has mild depressive symptoms and has reduced her food intake in the last 8 months. She exercises daily and admits to inducing vomiting occasionally after a meal. Her body mass index is 16.8 kg/m2. She has a blood pressure of 95/65 mmHg and a heart rate of 70 beats/minute. What is the most appropriate management?
Correct Answer : D
She falls into the category of moderate anorexia. Admission to the hospital is not warranted because her blood pressure and heart rate are fine and her BMI is still above 15.
Q.15. A 25 year old woman presents to the GP with low mood. She has an increased appetite and has gone up 2 dress sizes. She also complains that she feels very tired and often only gets out of bed in the afternoon despite sleeping early. What is the most likely diagnosis?
Correct Answer : C
Atypical depression is a subtype of major depression or dysthymic disorder that involves several specific symptoms, including increased appetite or weight gain, sleepiness or excessive sleep, and marked fatigue or weakness. Atypical depressive episode regarded as a subtype of depressive disorder, rather than a separate entity.
Clinical features :
• Mood is depressed but remains reactive
• Hypersomnia (sleeping more than 10 hours/day, at least 3 days / weeks, for at least 3 months)
• Hyperphagia (excessive eating with weight gain of over 3kg in 3 months)
• ‘Leaden paralysis’ (feeling of heaviness in the limbs).
Epidemiology : Onset is usually in the late teens and early twenties.
Q.16. A 30 year old woman comes to clinic in tears trying to describe the constant irritability she is in when dealing with her 2 small children. She describes herself as easilystartled. She is unable to concentrate for long but attributes it to poor sleep as she often gets nightmares of a house fire. Her husband died in a house fire while she was sleeping in the other room last year. What is the most appropriate management?
Correct Answer : C
This woman is suffering from post-traumatic stress disorder.
Fluoxetine is an SSRI.
SSRIs are used second to CBT. As CBT was not an option in this question, fluoxetine would be the next best choice.
Q.17. A 34 year old man who is a known schizophrenic has been going through a depressive phase for the past few months since the death of his father. During discussions with his siblings and friends, he is seen laughing and smiling inappropriately while talking about his father’s death. He does not seem to realise that his actions are inappropriate. What is the most appropriate term that describes his symptoms?
Correct Answer : A
Incongruent affect The term “incongruent” means out of place. Incongruent effect is sometimes seen in schizophrenia where their behaviour is out of place.
For example, talking about problems in their family while laughing or laughing when your pet dies. These behaviours are seen as not consistent with the patient’s current mindset. This is also seen in bipolar patients.
Q.18. A 23 year old woman has had several sudden onset episodes of palpitations, sweating, nausea and overwhelming fear. On one occasion she was woken from sleep and had the fear that she was going insane. She has no previous pyschiatric medical history and is not on any medication. What is the most likely diagnosis?
Correct Answer : B
It is quite difficult to differentiate generalized anxiety disorder from panic disorder. People with generalized anxiety disorder feel anxious most days and often struggle to remember the last time they felt relaxed. As soon as one anxious thought is resolved, another may appear about a different issue.
Panic disorder, however, is where you have recurring and regular panic attacks, often for no apparent reason similar to thisstem where she had several sudden onset episodes of panic.
Panic disorder is classified as having 2 recurrent panic attacks, which are not secondary to substance misuse, medical conditions, or another psychiatric disorder. The frequency of occurrence may vary from many attacks a day to only a few a year. Usually, patients have a persistent worry about having another attack or the consequences of the attack.
Symptoms/signs :
• Physical symptoms & signs related to autonomic arousal (e.g. tremor, tachycardia, tachypnoea, hypertension, sweating).
• Concerns of death from cardiac or respiratory problems may be a major focus, leading to patients presenting to emergency medical services.
Q.19. A 48 year old man states he is a well known professor teaching rocket science in a University. He states that the amount of people who attend are within the thousands and they would fly in from different parts of the world to hear his lectures. He states that he owns majority of United Kingdom and in a few years people would think of him as a god. What is the most likely disorder?
Correct Answer : A
This is a difficult question and many of the choices are potentially correct however the one disorder that stands out is delusion. A delusion is a mistaken belief that is held by a person with a strong conviction despite true evidence against the belief.
The subtype of delusion called “grandiose delusions” is likely what this man is suffering from. This subtype of delusions is seen in manic episodes of bipolar disorder, schizophrenics, and occasionally in substance abuse disorders.
The option of mania and bipolar disorder is not entirely wrong but delusion fits better as the answer.
Q.20. A 10 year old boy with behavioural problems is taken to the clinic by his parents. During the appointment, the boy barks and shouts expletives. He is constantly blinking his eyes and unable to sit still. What is the most likely diagnosis?
Correct Answer : D
This is a classic scenario for Tourette’s syndrome in the exam. Other clues that may appear on exam may be the child yelling in class intermittently or shouting expletives. Most Tourette’s syndromes are diagnosed at 6-8 years, maximum to the age of 13.
The other syndromes are less likely to be the answer:
Asperger syndrome : Characterized by severe persistent impairment in reciprocal social interactions, repetitive behaviour patterns, and restricted interests. IQ and language are normal or, in some cases, superior.
Although tics (like the above case) can also be found in Asperger's syndrome, it is more specific for Tourette’s syndrome. Not to mention, the question would include impairment of social skills if the examiners wanted you to have picked Asperger syndrome.
Cotard’s syndrome is a rare mental illness in which an afflicted person holds the delusion that they are dead Rett’s syndrome.
There is normal development for 2–3 years, followed by a loss of acquired motor, language, and social skills between ages 3 and 4yrs. Stereotypes and compulsions are common.
Ekbom’s syndrome. Also called restless leg syndrome. Unpleasant, often painful sensations in the legs, particularly on sleep onset.
Q.21. A 19 year old man accuses his friend of making his right arm swing out to hit a stranger at the park. There is no evidence of this as his friend was at home at that time. What is the most appropriate term to describe this condition?
Correct Answer : D
Delusion of control is the false belief that another person, group of people, or external force controls one's general thoughts, feelings, impulses, or behavior.
Q.22. A 44 year old man is very depressed and miserable after wife’s death 6 months ago. He sees no point in living now that his wife is not around. He feels regret and wishes he never existed. He refuses any medical help offered. His son has brought him to the emergency department. The son mentions that he cannot deal with the father’s depression any longer as he has a job and family to attend to. What is most appropriate next step?
Correct Answer : B
This patient is refusing any help offered thus voluntary admission to psychiatric wards is not possible. Compulsory admission under the Mental Health Act is the most appropriate answer.
The Mental Health Act allows people with a ‘mental disorder’ to be admitted to a hospital, detained, and treated without their consent – either for their health and safety or for the protection of other people.
Q.23. A 30 year old man who served in the army 6 months ago presents with lack of interest in enjoyable activities and feeling low. He often wakes up in the middle of the night because of nightmares of gun fire. He feels irritable and has difficulty concentrating. He tries not to watch the news as it reminds him of war. What is the most appropriate initial therapy?
Correct Answer : C
This man is suffering from post-traumatic disorder Post-traumatic disorder (PTSD) develops following a traumatic event. In this case, it was a war that stimulated PTSD.
Q.24. A 14 year old girl was found by her mother collapsed on the floor at home and brought to the Emergency Department. She was found on the floor at home but regained consciousness just before the paramedics arrived. She gives a history of increasing fatigue and intolerance to cold over the past few weeks since joining the gym 2 months ago. On further questioning, her mother says that she found laxatives and diuretics in her daughter’s room and is concerned abut it being the cause of her collapse. She denies her weight being an isue despite having a body mass of 15 kg/m2. She has a systolic blood pressure of 85 mmHg and a heart rate of 55 beats/minute. During examination, it is noted that she finds it difficult to get up from a lying position without using her hands. What is the most likely reason for her symptoms?
Correct Answer : A
This girl has clinical features of anorexia nervosa. Over exercising, diuretics and laxatives over several weeks could cause a rapid weight loss which has detrimental effects to the cardiovascular system leading to syncope.
Young girls with anorexia nervosa often feel fatigue and intolerance to cold due to their low BMI. Their muscle power is decreased so they may find it more difficult to sit up without using their hands to support them.
Q.25. A 30 year old woman complains of feeling restless, muscle tension and sleep disturbance on majority of the days over the last 9 months. She worries excessively about a number of everyday events and activities and is unable to control these feelings which are impairing her ability to hold down her job. What is the most likely diagnosis?
Correct Answer : A
There is a fine line between Generalised Anxiety Disorder (GAD) and Panic attacks. They both can present similarly. The major difference is panic attacks are usually short-lived (lasting no longer than 20–30min (rarely over 1 hour). ‘Excessive worry and feelings of apprehension about everyday events like in this case point towards the diagnosis of GAD.
Q.26. A 64 year old man believes a female newscaster on the television is communicating directly to him when she turns a page and when she looks at the camera. What is the most likely type of delusion that this man is suffering from?
Correct Answer : D
Delusion of reference is the false belief that insignificant remarks, events, or objects in one's environment have personal meaning or significance. For example, someone constantly gives him or her special messages through the newspaper.
Q.27. A 29 year old man has been severely depressed over the last 3 years now believes that he does not exist and never existed in this world. He has poor eye contact and speaks softly. He says that people around him are unable to listen and see him because he is inaudible and invisible. What kind of delusion is he suffering from?
Correct Answer : A
Nihilistic delusions are the delusional belief that the patient has died or no longer exists or that the world has ended or is no longer real. Nothing matters any longer and continued effort is pointless. It is a feature of psychotic depressive illness.
Patients may believe that he/she is dead and may ask people to bury them.
Q.28. A 64 year of man has recently suffered from a myocardial infarction 5 months ago. He has been having trouble sleeping and seems depressed. His regular medications include aspirin, atorvastatin and ramipril. What is the most appropirate medication to start him on?
Correct Answer : C
For the majority of patients with moderate depression, selective serotonin reuptake inhibitors (SSRIs) are considered first-line. If the question gives you options for SSRIs, sertraline would be the best answer to pick as sertraline has a good safety profile with patients with myocardial infarction.
Q.29. A 38 year old man has disturbing thoughts about his house being infected by germs. He is anxious about safety and checks the locks of his doors repeatedly before going to bed. He has been washing his hands every time he touches the lock. This can be 5 go 10 times an hour. What is the most appropriate management?
Correct Answer : D
The diagnosis here is obsessive-compulsive disorder (OCD). Exposure and response prevention (ERP) is included in cognitive behavioural therapy (CBT) in the treatment of those who present with OCD. The method is predicated on the idea that therapeutic effect is achieved as subjects confront their fears and discontinue their escape response.
In this case, the patient would be exposed to his feared stimulus and would refuse to respond with any safety behaviors. SSRIs are also a treatment choice for OCD. However as this is only a mild functional impairment, ERP would be a more appropriate answer.
Q.30. A 33 year old woman is brought in to the Emergency Department with coarse tremors, nausea, vomiting, and abdominal pain. She is known to have bipolar affective disorder and is currently taking 600 mg of lithium carbonate twice a day. She has been feeling lethargic for the past week. She has a temperature of 38.1 C and a heart rate of 85 beats/minute. On examination, she looks drowsy with impaired concentration. Her blood results show: Sodium 140 mmol/L Potassium 4.5 mmol/L Urea 7 mmol/L Creatinine 140 micromol/L eGFR 60 Calcium 2.9 mmol/L What is the most likely cause of her presentation?
Correct Answer : A
Lithium is still used for bipolar disorder and has been proven to be a very effective treatment. In this stem, we see the blood that shows hypercalcaemia. It is important to remember that lithium toxicity can cause hypercalcemia. Medical professionals who deal with lithium monitoring are trained to look out for side effects of lithium toxicity such as severe or coarse hand shaking or tremors, blurred vision, abdominal pain with diarrhoea, slurred speech, muscle twitching, confusion, and muscle weakness.
We can see some of the symptoms here in this stem. Patients need regular blood tests to monitor lithium levels every 3 months and regular liver function tests and urea and electrolytes every 6 months.
Q.31. A 52 year old woman has been depressed ever since her husband died half a year ago. She was started on amitryptiline by her GP 3 months ago to help battle her depression. She now feels much better and sleeps well. She still thinks about her husband occasionally and the thoughts bring her mood down but she has drastic improvements as compared to a few months ago. She wants to know if she can stop medication. What is the best advice to give her?
Correct Answer : D
Continuing therapy for at least 6 months is advised to reduce the risk of relapse even when patients are feeling “better”. Patients should be reassured that antidepressants are not addictive.
Depression is an important feature of bereavement. We note a good response with this lady as she feels better and sleeps well, but antidepressive therapy should still be continued for another 3 months (6 months in total) to reduce the risk of relapse.
Q.32. A 21 year old woman was brought to the Emergency Department by her boyfriend. She has many self inflicted superficial lacerations on her forearm. She is distressed and constantly says her boyfriend is going to end the relationship. She denies trying to end her life. What is the most likely diagnosis?
Correct Answer : A
Borderline Personality Disorder usually characterized by mood swings, marked impulsivity, unstable relationships, and inappropriate anger. They can be very dramatic. They are usually attention-seekers and may have multiple self-inflicted scars. They may threaten to commit suicide.
Q.33. A 22 year old man finds it difficult to come out of a room without having to turn the light switch off and on 3 minutes. He has tried more than several times to go out of the room without having to do this particular compulsion however he still returns to the room feeling agitated that it was not done. He recognizes that he has a problem and is willing to see a psychiatrist. What is the most accurate term that describes this situation?
Correct Answer : D
Insight In psychology and psychiatry, insight can mean the ability to recognize one's mental illness. This form of insight has multiple dimensions, such as recognizing the need for treatment and recognizing the consequences of one's behavior as stemming from an illness.
For example, people with obsessive-compulsive disorder and various phobias tend to have relatively good insight that they have a problem and that their thoughts and/or actions are unreasonable, yet are compelled to carry out the thoughts and actions regardless.
In general, a patient with insight is a patient who knows that their symptoms represent abnormality and seeks their diagnosis and appropriate treatment.
Q.34. A 32 year old woman complains of abdominal pain, shortness of breath, palpitations, unsteadiness, and numbness of the lower limbs. These symptoms have been on going for several months. An ECG, chest x-ray, abdominal ultrasound and neurological exam has returned with normal results. What is the most likely diagnosis?
Correct Answer : A
Somatization disorder is the experience of bodily symptoms with no physical cause for them, with presumed psychological causation. All investigations would be normal.
Hypochondriasis is the persistent belief in the presence of an underlying serious DISEASE, e.g. cancer or HIV. The patient again refuses to accept reassurance or negative test results.
Munchausen's syndrome is also known as factitious disorder. Patients intentionally falsify their symptoms and past history and fabricate signs of physical or mental disorders with the primary aim of obtaining medical attention and treatment.
The diagnostic features are the intentional and conscious production of signs, falsification, or exaggeration of the history, and the lack of gain beyond medical attention and treatment.
Q.35. A 38 year old man was brought to the hospital by his sister because he inflicted self harm earlier in the day. His sister says that he has been depressed for about a year since his girlfriend left him. He lost his job last month and feels that there is nothing worth living. The patient started experiencing suicidal thoughts about a month ago and two days ago he began to prepare a note for his suicide. His sister is concerned about his mental health as he had asked her to help him end his life earlier that day. She became extremely concerned after he had cut himself and convinced him to go with her to the hospital. He is seen to have multiple lacerations in both arms with multiple healed scars along the arm and forearm. During the consultation, the patient was noted to be in a state of self-neglect and had poor eye contact. He was listless with poverty of speech and his speech had a low volume. What is the most appropriate course of action?
Correct Answer : B
Admission should be to a ward where close observation and monitoring are possible, whenever there is a significant risk of harm to self (or others). The patient depicted in this scenario has severe depression. He has obvious depressive symptoms with severe functional impairment (self-neglect).
In addition, he has begun to actively plan a suicide attempt. This is a red flag. If you have reason to believe that he will harm himself or others, you may detain him and admit him.
Mirtazapine is an atypical antidepressant. It is not the correct answer for this scenario. You cannot just give this patient an antidepressant and send him home because he is at high risk of suicide. In addition, antidepressants take weeks to take effect and this patient needs help now.
Electroconvulsive therapy (ECT) is a very controversial mode of treatment. The only people who would be deciding to administer electroconvulsive therapy would be specialists and consultants.
Q.36. A 64 year old man has just suffered from a myocardial infarction. Before discharge, he seems to be in a sad mood and avoids eye contact. The nurses report that he has skipped his last two meals. What is the best treatment for this man’s condition?
Correct Answer : D
For the majority of patients with moderate depression, selective serotonin reuptake inhibitors (SSRIs) are considered first-line If the question gives you options for SSRIs, sertraline would be the best answer to pick as sertraline has a good safety profile with patients with myocardial infarction.
Q.37. A 62 year old man who was admitted for surgery 3 days ago suddenly becomes confused. His attention span is reduced. He is restless and physically aggressive and picks at his bed sheets. What aspect of the patient’s history recovered in his notes is most likely to aid in making the diagnosis?
Correct Answer : A
Abstinence from alcohol in the hospital can cause delirium tremens which is noted here by his sudden confusion, restlessness, and physical aggressiveness.
Delirium tremens :
• Delirium tremens usually begin 24-72 hours after alcohol consumption has been reduced or stopped.
• Seen in chronic alcoholics
• The symptoms/signs differ from usual withdrawal symptoms in that there are signs of altered mental status. These can include hallucinations (auditory, visual, or olfactory), confusion, delusions, and severe agitation. Seizures can also occur.
A common scenario would be a chronic alcoholic who is admitted for an operation where that patient would have to be abstinent from alcohol. A few days after being abstinent, he or she would start having symptoms of an altered mental status which include hallucinations.
Q.38. A 38 year old woman with episodes of mania followed by depression was started on medication. Improvement was seen and she no longer complains of these episodes. What is the most likely medication that she was started on?
Correct Answer : A
Please see Q-3
Q.39. A 62 year old retired school teacher has had a repair of strangulated femoral hernia 3 days ago. She has become agitated, aggressive and confused over the past 12 hours. She has developed auditory hallucinations. Her blood tests show: Haemoglobin 129 g/L Mean cell volume (MCV) 112 fL Gamma-glutamyl transferase (GGT) 120 u/L Alkaline phosphatase (ALP) 110 iu/L What is the most appropriate management?
Correct Answer : A
Abstinence from alcohol in the hospital can cause delirium tremens. The chronic alcoholism is supported by high MCV and GGT here. The GGT result is more useful than the MCV result as a “red flag” to raise the suspicion that the person is drinking too much. Symptoms fit delirium tremens. Seizures can be seen in delirium tremens.
It is unlikely to be Wernicke’s encephalopathy or Korsakoff’s psychosis as in exam they would usually have a triad of confusion, ataxia, and ophthalmoplegia.
Q.40. A 20 year old boy is brought by his parents to the emergency department. He is agitated and euphoric. The parents suspect that he has taken drugs. Examination reveals a perforated nasal septum. What is the most likely drug that was taken?
Correct Answer : B
This is a high-yield question in the exam. The perforated nasal septum is a huge clue that the route of intake is nasally inhaled. In the majority of cases, the answer when you see that term is cocaine. This along with the signs and symptoms of euphoria and agitation points towards cocaine.
Q.41. A 30 year old man complains of episodes of hearing music and threatening voices within a couple of hours of heavy drinking with his friends at a friend’s birthday party. What is the most likely diagnosis?
Correct Answer : D
Alcohol hallucinosis can occur during acute intoxication or withdrawal. It usually appears 12-24 hours after alcohol has stopped. It involves auditory and visual hallucinations, most commonly accusatory or threatening voices.
The answer is unlikely to be delirium tremens because of the time frame in which he was drinking alcohol. Delirium tremens usually happens days after alcohol consumption has been stopped and he would need to be a chronic alcoholic.
Q.42. A 21 year old woman has had several sudden onset episodes of palpitations, sweating, nausea and overwhelming fear. On one occasion, she was woken from sleep and feared she was going insane. She has no previous psychiatric disorder. What is the most likely diagnosis?
Correct Answer : B
Please see Q-18
Q.43. An 82 year old schizophrenic man is brought to the Emergency Department in an agitated state. He is lashing out violently and throws any equipment around him at the hospital staff. He seems to be hearing voices which are causing him distress. He does not have any intravenous access. Which is a drug of choice in the management of agitation in this man?
Correct Answer : C
In acute psychosis, the drug of choice is haloperidol. It is still considered a first-line medication for rapid tranquilization. Haloperidol is a drug of choice for the management of psychotic episodes particularly in the elderly.
Diazepam is a long-acting benzodiazepine and so should not be used intramuscularly for rapid tranquilisation as it has a prolonged and erratic absorption. Besides, benzodiazepines generally are not recommended in elderly patients. Clonazepam is a long-acting benzodiazepine and can only be given orally which makes it inappropriate in this situation.
Q.44. A 22 year old woman was brought to the A&E by her friends. She presents with tremors. On examination, she was found to have a temperature of 37.4 C and her pupils dilated. She says when she closes her eyes, she can see colours. What is the most likely drug that she has taken?
Correct Answer : B
The immediate clue here is that she sees colours when she closes her eyes. These are the effects of LSD.
LSD can cause visual hallucinations, agitation, excitement, tachycardia, and dilated pupils.
You would need to admit this patient to a safe environment and provide symptomatic treatment of the agitation such as benzodiazepines. If the overdose is severe enough, she may go into a coma or have respiratory arrest which needs to be treated supportively.
Q.45. A 29 year old woman has been taking selective serotonin reuptake inhibitors for the past 6 months for depression after the death of her husband 10 months ago. She feels her symptoms have improved and has decided to stop her medications. Several weeks after discontinuing her medications, she feels she has developed pancreatic cancer similarly to her late husband. What is the most appropriate next step in management?
Correct Answer : B
This woman requires a psychiatric referral i.e. Neuropsychiatric analysis. She is having a hypochondriacal delusion. Hypochondriasis can be defined as the persistent belief in the presence of an underlying serious disease, e.g. cancer or HIV.
Examples :
• A minor headache is caused by a brain tumour
• A mild rash is the start of skin cancer
• Tiredness is caused by HIV.
Reassurance by a doctor will not help a hypochondriac as a hypochondriac would fear that the doctor has just not found evidence of a serious disease yet.
Q.46. A 44 year old alcoholic was admitted for alcohol intoxication. He has been treated and he is now planned for discharge. He admits to not being able to spend a day without drinking. Which of the following statements would show that this man is still dependant on alcohol?
Correct Answer : D
This question is testing your knowledge of the AUDIT questionnaire. There are many alcohol screening tools and questionnaires but the AUDIT questionnaire remains one of the tops and a must-know for the exam.
Q.47. A 28 year old woman complains of hearing strange voices in her bedroom as she is falling asleep in the night. She says there is no one in the room except for her. She is otherwise healthy and without mental illness. What is the most likely diagnosis?
Correct Answer : B
Hypnagogic hallucination is a transient false perception experienced while on the verge of falling asleep (e.g. hearing a voice calling one’s name which then startles you back to wakefulness to find no one there).
The same phenomenon experienced while waking up is called hypnopompic hallucination. Frequently experienced by healthy people so not a symptom of mental illness.
Q.48. A 38 year old woman presents with palpitations and sweating for most of the day throughout the year. She feels restless and unable to sleep well. She is irritable and finds it difficult to concentrate. She has occasional chest discomfort when thinking about everyday events. She reports no triggering factor. There is no previous psychiatric illness. On examination, her blood pressure is 120/80, pulse is 85 beats/minute. What is the most likely diagnosis?
Correct Answer : A
There is a fine line between Generalized Anxiety Disorder (GAD) and Panic attacks. They both can present similarly. The major difference is panic attacks are usually short-lived (lasting no longer).
Q.49. A 35 year old man is seen by his psychiatrist for severe depression. He says that the world has ended and is no longer real. He thinks that he no longer exists in this world. He barely has eye contact with the psychiatrist. What is the most appropriate diagnosis?
Correct Answer : C
Nihilistic delusions are the delusional belief that the patient has died or no longer exists or that the world has ended or is no longer real. Nothing matters any longer and continued effort is pointless.
It is a feature of psychotic depressive illness. Patients may believe that he/she is dead and may ask people to bury them.
Q.50. A 33 year old female presents to her GP because of low moods. She has difficulty sleeping and feels tired at work. She refuses to go out with her friends and rather spends time resting on her bed. She is eating less and has lost 8 kg in the last 10 weeks. A year ago, she was productive, full of energy, optimistic, needed very little sleep and always wanted to go out. Her BMI is 27. What is the most likely diagnosis?
Correct Answer : B
This stem portrays a good history of depression with a background of mania in the past. The symptoms of mania in this stem are very subtle and somewhat within normal limits but the real giveaway is when there is reduced need for sleep. This would be a classic scenario when they would like you to pick Bipolar disorder.
Q.51. A 48 year old woman who is always socially withdrawn has stopped going out of the house. It initially started when she felt distressed whenever she ate in front of strangers at work as she felt they were staring at her. This soon progressed to affect her work as she started to avoid business meetings because she fears that people will criticize her. What is the most likely diagnosis?
Correct Answer : B
The abnormal excessive fear of being criticized is seen in social anxiety disorder where sufferers would avoid these social events including business meetings.
SOCIAL ANXIETY DISORDER (SOCIAL PHOBIA) : Social anxiety disorder is the persistent fear and anxiety about one or more social or performance situations. It is characterized by marked fear of one or more social or performance-related situations where the person is exposed to scrutiny and in which embarrassment may occur.
Q.52. A 38 year old woman believes that a famous politician has been sending her flowers every day and is in love with her. She says that he drops hints that he loves her when he speaks publicly on television. The famous politician has had no contact with this lady. What is the most likely diagnosis?
Correct Answer : B
Erotomaniac stalkers have the delusional belief that another person, usually of higher social status, is secretly in love with them. The sufferer may also believe that the subject of their delusion secretly communicates their love through seemingly innocuous acts, or if they are a public figure through clues in the media.
The object of the delusion usually has little or no contact with the sufferer, who often believes the object initiated the fictional relationship. Erotomaniac delusions are typically found as the primary symptom of a delusional disorder and in schizophrenia or mania.
Q.53. A 27 year old man presents with symptoms characterized by alternating mood swings associated with flight of ideas, and overactivity. Three months ago, he had low moods with lack of energy. What is the most likely diagnosis?
Correct Answer : D
It is clear that this patient is having periods of depression with other periods of mania. Flight of ideas is thought that jumps from topic to topic. It is the subjective experience of one’s thoughts being more rapid than normal. Meaningful connections between thoughts are maintained. It occurs especially in the manic phase of bipolar disorder.
Q.54. A 71 year old woman looks disheveled, unkempt and with poor eye contact. She has recently lost her husband 2 months ago. She feels hopeless and has been tearful all week. Which option describes her condition?
Correct Answer : C
She has symptoms of depression. Depressive symptoms One of the most common referrals in liaison psychiatry is of patients with low mood. Patients may present with low mood, tearfulness, hopelessness regarding recovery, biological depressive features (poor sleep, appetite, energy, and concentration).
Q.55. A 65 year old woman thinks she has died 3 months ago and is very distressed that nobody has buried her up till now. She hears people’s voices which tell her that is evil and needs to be punished. She barely has any eye contact when speaking to the health care professionals. Her previous visits have included symptoms of feeling of guilt and personal inadequacy. What is the most likely explanation for her symptoms?
Correct Answer : D
Some people who have severe clinical depression will also experience hallucinations and delusional thinking, the symptoms of psychosis. Nihilistic delusions are the delusional belief that the patient has died or no longer exists or that the world has ended or is no longer real. Nothing matters any longer and continued effort is pointless.
It is a feature of psychotic depressive illness. Patients may believe that he/she is dead and may ask people to bury them.
Q.56. A 33 year old schizophrenic man hears people only when he is about to fall asleep. What is the most likely phenomenon?
Correct Answer : C
Hypnagogic hallucination is a transient false perception experienced while on the verge of falling asleep (e.g. hearing a voice calling one’s name which then startles you back to wakefulness to find no one there). The same phenomenon experienced while waking up is called hypnopompic hallucination. Frequently experienced by healthy people and so not a symptom of mental illness.
Q.57. A 37 year old woman was admitted for a femur frature repair after a road traffic accident. On the fourth day post-op she becomes confused and starts picking on her bed sheets. She complains of seeing spiders all over her bed. What is the most likely diagnosis?
Correct Answer : A
Withdrawal of alcohol due to hospital admission often leads to delirium tremens. This is noted here by her confusion and picking on her bed sheets. This is seen a couple of days after consumption of alcohol is stopped as seen in this question.
Wernicke's encephalopathy is unlikely as it presents with a triad of confusion, ataxia, and ophthalmoplegia. Korsakoff’s psychosis is characterized by the addition of Antero- and retrograde amnesia and confabulation in addition to the above classic triad of Wernicke's encephalopathy.
Q.58. A 38 year old man keeps having intrusive thoughts about having dirt under the bed. He cannot keep himself from thinking about these thoughts. If he tries to resist, he starts having palpitations. What is the most likely diagnosis?
Correct Answer : C
Please see Q-11
Q.59. A 22 year old woman is brought to the local hospital by her mother with the primary complaint of recurrent depressive episodes. Her main symptoms during her depressive episodes are low mood, anhedonia, excessive somnolence, and weight loss secondary to poor appetite. Recently however, she has become overly happy and rarely sleeps. Her mother complains that she has mixed out all her credit cards and that she stays out all night drinking and partying. Some nights, she doesn’t even come home. She has had multiple sexual partners during the past week and her mother expresses a concern about contraception. The patient was also recently involved in an altercation with the police. Her grandmother passed away two days ago and the mother tearfully describes her daughter laughing loudly during her grandmother’s funeral. What is the best descriptor for this patient’s behaviour?
Correct Answer : D
In order to answer this question appropriately, we need to figure out what is the primary cause of this patient’s symptoms. This patient has bipolar disorder. Bipolar disorder is characterized by episodes of low mood alternating with episodes of high mood.
Mood incongruence is a term used to describe a psychotic feature of bipolar disorder wherein the person’s belief or action does not match his or her mood.
An example would be laughing at the death of a loved one or believing that you have superpowers while going through a depressive episode. Mood congruence also describes a psychotic symptom of bipolar disorder, but, in this case, the belief or action is consistent with that person’s mood.
For example, feeling suicidal when a loved one dies or believing that you have superpowers when going through a manic episode.
Even though mood congruence and incongruence both describe psychotic features of bipolar disorder, psychosis is not correct as the question asks for the best descriptor of the patient’s behavior.
Q.60. A 35 year old chronic alcoholic has been trying to stop his alcohol drinking habit. He has been going for support meetings. He wants to know if there is a medication that can help reduce his alcohol cravings. What is the most appropriate medication?
Correct Answer : B
Acamprosate is used in combination with psychological intervention in moderate to severe alcohol dependence. It reduces the risk of relapse by reducing the cravings of alcohol. It is started after a successfully assisted withdrawal. Other alternatives to acamprosate which are not mentioned here are naltrexone or nalmefene.
Q.61. A 28 year old woman has episodes of perioral tingling and carpopedal spasms every time she has to give a public talk. These symptoms also happen to her before interviews, exams and after arguments. She describes these episodes as short lasting only a couple of minutes but with intense fear. What is the most appropriate management to prevent further episodes?
Correct Answer : C
Cognitive behavioural therapy (CBT) should be used to treat panic disorders as there is really good evidence to support its use. Benzodiazepines should not be prescribed for the treatment of individuals with panic disorder as they have poor outcomes in the long term. Rebreath into paper bags is useful in acute settings but not as prevention.
Q.62. A 28 year old business man returned from a trip to Kenya 2 years ago. He attends a sexual clinic worried that he has contracted HIV. Antibody screening test for HIV has come back negative. There were 2 similar HIV tests performed 6 months and 9 months ago which have both come back as negative. This is his 5th visit to the clinic claiming that he is HIV positive. What is the most likely diagnosis?
Correct Answer : B
Hypochondriasis is the persistent belief in the presence of an underlying serious disease, e.g. cancer or HIV. The patient again refuses to accept reassurance or negative test results.
Q.63. An 18 year old girl with a body mass index of 17.8 kg/m2 has bilateral parotid swelling with thickened calluses on the dorsum of her hand. What is the most likely diagnosis?
Correct Answer : A
This question is not entirely specific. Usually, for a bulimia nervosa question to appear, the stem would give more of a history which includes either fasting or intensive exercise. Self-induced vomiting would be too obvious for the question writers to give. Bilateral parotid swelling and thickened calluses on the knuckles from self-induced vomiting may be found on examination in bulimia nervosa.
Q.64. A 33 year old man with a family history of panic disorder has palpitations, tremors, sweating and muscles tightnes on 3 occasions in the last 6 weeks. His pulse rate is 85 beats/minute and his blood pressure is 120/80 mmHg. What is the most appropriate long-term treatment for him?
Correct Answer : C
He is suffering from panic disorder. An SSRI or cognitive behavioural therapy would be appropriate. Sertraline is an SSRI.
Q.65. A 33 year old man attends his appointment with the psychiatrist. He says that he is no longer alive. He wants his family to bury him. What is the most appropriate diagnosis?
Correct Answer : C
Nihilistic delusions are the delusional belief that the patient has died or no longer exists or that the world has ended or is no longer real. Nothing matters any longer and continued effort is pointless. It is a feature of psychotic depressive illness. Patients may believe that he/she is dead and may ask people to bury them.
Somatization disorder : The experience of bodily symptoms with no physical cause for them, with presumed psychological causation.
Q.66. A 29 year old man has been found in the park, drunk and brought to the emergency department by ambulance. He recently lost his job and had a divorce 3 months ago. He has intense feelings of feeling worthless and being a failure. He also hears voices telling him he is worthless. What is the most likely diagnosis?
Correct Answer : A
This affected individual has lost his job and had a recent divorce. Depression should be something to consider. The auditory hallucinations are signs that this man is having some form of psychosis. The most probable diagnosis here is psychotic depression. Feeling worthless is in line with the delusions of guilt that are seen in psychotic depression.
Q.67. A couple has just finished their detox regime and wants a drug with a pharmacological action to serve as a deterrant when they take alcohol. What is the most approprie medication to start?
Correct Answer : A
Disulfiram acts as a deterrant. It reacts with alcohol and causes the patient to feel flushed, nauseous, dizzy, and faint. Because of these effects, patients would stay away from alcohol. Disulfiram works so well that it can even react with alcohol found in perfumes and aerosol sprays causing similar symptoms.
Q.68. A 19 year old female is brougthto the hospital by her parents. They are concerned about her weight. Her body mass index is 12.1 kg/m2. She has a mildly depressed mood and has low self-esteem. She has amenorrhoea. She has reduced her food intake in the past couple of months. She has a blood pressure of 70/50 mmHg and a heart rate of 44 beats/minute. What is the most appropriate management?
Correct Answer : D
She is suffering from anorexia nervosa. Her BMI is critically low medical admission is warranted to provide her with proper nutrition. A pulse rate lower than 45 beats/minute and hypotension raises great concern and is definitely a criterion for hospital admission. The most common cause of death in patients with anorexia nervosa is due to cardiac complications.
This is why they are admitted to medical wards and not psychiatry wards when the condition is severe.
Q.69. A 25 year old woman has had several sudden onset episodes of palpitations, sweating, and fear. She notices her hands shake when they occur. These episodes occur almost everyday and sometimes can wake her from her sleep. She has no previous psychiatric disorder and is not on any medications. What is the most likely diagnosis?
Correct Answer : B
Please see Q-18
Q.70. A 32 year old woman is brought to the Emergency Department by ambulance in an agitated state. The ambulance crew gives you the handover that her neighbours had seen her roaming around the streets throwing money and shouting she is going to die. She refuses to let any health care come close to her to perform observations or to take her bloods. She is unable to answer any of yur questions. At the end of every question, she laughs hysterically and repeatedly shouts that she smokes, drinks and walks amongst alients. It is deemed that she does not have capacity to make decisions and a compulsory admission under Mental Health Act is performed. She continues to be agitated and attempts to run away from the hospital but is stopped by security. Which is the most appropriate medication to administer?
Correct Answer : B
This patient is suffering from acute psychosis. No matter the reason, once she is sectioned, sedation would be the next step to calm her down for further investigations such as observations and blood tests.
There are a few drugs used for rapid tranquilization.
Guidelines recommends using lorazepam, olanzapine, or haloperidol. In this context, it is usually appropriate to administer intramuscular lorazepam 2 mg.
If she continues to be agitated, she may require haloperidol intramuscularly.
The reason lorazepam is preferred over the other benzodiazepines is that it is short-acting with a short half-life and respiratory depression is unlikely with the prescribed doses. Diazepam on the other hand should not be used intramuscularly for rapid tranquilisation as it has prolonged and erratic absorption.
Q.71. A 22 year old man was found overdosed on heroin. He has decreased respiratory rate and has lost consciousness. What is the most appropriate management?
Correct Answer : C
Opiate overdose is treated with naloxone. It is given intravenously and repeated every 2 minutes until the patient’s breathing is adequate.
Q.72. A 23 year old man comes to the emergency department with a history of drug misuse. He recognizes that he has a problem and is willing to see a psychiatrist. What is the most accurate term that describes this situation?
Correct Answer : B
Please see Q-33
Q.73. A 26 year old political refugee has sought asylum. He complains of poor concentration. He keeps getting thoughts of his family whom he saw was killed in a political coup. He is unable to sleep well and feels hopeless. In the past mnth, he has had 3 brief episodes lasting a few minutes of feeling detached from his surroundings, as if he was watching the world through misty glass. He is worried that he is going mad. What is the most likely diagnosis?
Correct Answer : C
The phenomenon where he feels detached from his surroundings as if he is an observer of himself is called depersonalization.
It is a phenomenon where subjects feel that the world has become dreamlike and vague. It is seen in individuals who have experienced severe trauma. It can be one of the symptoms of post-traumatic stress disorder (PTSD).
Q.74. A 28 year old woman presents with feeling anxious for most days throughout the past 6 months. She feels distress when thinking about money and her job. She is unable to sleep well and finds it difficult to concentrate at work. On occasion, she panics and she feels she is about to die. On examination, her blood pressure is 120/80, pulse rate is 90 beats/minute. What is the most appropriate medication to prescribe?
Correct Answer : A
The patient is suffering from generalized anxiety disorder. If medications were to be prescribed, the first line would be an SSRI.
Q.75. A 30 year old woman who has been feeling low and having difficulty in concentrating since her mother passed away 2 months ago. She feels lethargic and has been crying more often lately. What is the most likely diagnosis?
Correct Answer : C
An adjustment disorder occurs when an individual is unable to adjust to or cope with a particular stress or a major life event.
Q.76. A 19 year old college student has a firm and unshakable belief that he is being followed by terrorists who are plotting against him. He says they follow him wherever he goes. What is the most appropriate term for his condition?
Correct Answer : A
Persecutory delusion is a delusional belief that one’s life is being interfered with in a harmful way. It refers to false beliefs or perceptions in which a person believes that they are being treated with malicious intent, hostility, or harassment despite significant evidence to suggest otherwise.
This may occur in the context of being tormented, followed, or spied on.
Q.77. A 23 year old man feels anxious and agitated when faced with stress. He has an interview in 3 days and would like some help in relieving his symptoms for the interview. What is the most appropriate management?
Correct Answer : D
Medication usually is not used to treat phobias. However, it is sometimes prescribed to help people cope with the effects of anxiety. Beta-blockers are commonly used. It would be appropriate here as it is only for the short term.
Q.78. A 43 year old woman presents with low mood, and loss of libido. She feels tired all day and she attributes this to the fact that she wakes up 3 hours sooner than usual. She feels like she has been gaining weight. She also finds it difficult to concentrate for long periods of time. What is the most likely diagnosis?
Correct Answer : C
Symptoms of depression :
• Present for at least 2 weeks and represent a change from normal.
• Depressed mood: present most of the day, nearly every day, with little variation
• Anhedonia: markedly diminished interest or pleasure in all, or almost all, activities most of the day
• Weight change: loss of weight when not dieting or weight gain associated with decreased or increased appetite
• Disturbed sleep: Insomnia
• Fatigue or loss of energy.
• Reduced libido.
• Feelings of worthlessness or excessive or inappropriate guilt
• Diminished ability to think or concentrate or indecisiveness.
• Recurrent thoughts of death or suicide.
Q.79. A 32 year old man thinks nurses are plottingto harm him and are stealing his ideas straight out from his mind. Sometimes he feels the nurses are able to control his body. What is the most likely diagnosis?
Correct Answer : A
The patient is suffering from schizophrenia. This can be shown by the fact that he thinks that ideas are being stolen from his mind (thought withdrawal) and that the nurses are plotting to harm him (persecutory delusion). The feeling of nurses controlling his body is consistent with schizophrenia. It is called a passive phenomena.
Q.80. A 43 year old man attends the GP clinic complaining that his arm is dead and rotten and he wants it removed. He has very poor eye contact and stares at the floor majority of the consultation. On physical examination, the arm looks normal. What is the most appropriate diagnosis?
Correct Answer : A
Given in the previous questions
Q.81. A 26 year old woman has a history of bipolar disorder for 10 years and is taking lithium for it. She has been symptom free for the past 4 years. She is now planning her pregnancy and wants to know whether she should continue to take lithium. What is the most appropriate advice?
Correct Answer : C
Despite problems with tolerability, lithium still remains the gold standard in the treatment of bipolar affective disorder.
Lithium is a teratogen. If a woman taking lithium is planning a pregnancy and is well and not at high risk of relapse, she should be advised to stop taking the drug because of the risk of cardiac malformations in the fetus. This should be done by gradual discontinuation before conception.
Q.82. A 16 year old boy has been performing poorly in school since moving to a new school. He lacks interest and has very little social interactions. He has very few friends and finds it difficult to express himself. He prefers solitary activities. His teachers often complained that he is seen organising his stationaries on his table while counting to twenty each time and if disrupted, he would be very upset and anxious. One of his interests is collecting toy cars. He has over 2,000 toy cars and often spends hours lining them up. What is the most likely diagnosis?
Correct Answer : B
This boy displays traits of autism. He has difficulties in communication and forming relationships alongside having repetitive and obsessive behaviour that brings anxiety when disturbed. Patients with autism usually present at particular times of change. The change in this stem is represented by moving to a new school.
Q.83. A 20 year old man complains his movements are being watched. He feels as though his actions are being controlled by the radio. At times he hears voices describing his actions. What is the most likely diagnosis?
Correct Answer : D
He describes his movements as being controlled by the radio. This is known as a passive phenomena. He also describes voices describing what he is doing. This is called third-person auditory hallucinations. Sometimes these voices can be heard like a running commentary. For example, the patient hears one or more voices providing a narrative of their current actions, ‘he’s getting up…now he’s going towards the window’. All these features are features of schizophrenia.
Q.84. A 45 year old woman has been extensively investigatted for a lump she believes to be cancer. All investigations done show that the lump is unlikely to be cancer however she is not convinced and does not think the doctors are taking her seriously. She has demanded for another referral. What is the most appropriate term that describes her condition?
Correct Answer : C
Her obsession and persistent belief that there is an underlying serious disease is evidence of hypochondriasis. As there is a lump and she is not falsifying the symptoms, it is not Munchausen's syndrome or malingering. Hypochondriasis is the persistent belief in the presence of an underlying serious DISEASE, e.g. cancer or HIV. The patient again refuses to accept reassurance or negative test results.
Munchausen's syndrome is known as a factitious disorder.
Q.85. A 33 year old woman in the psychiatric ward diagnosed with schizophrenia, complains that she is unable to think straight because the nurse is stealing her thoughts. What is the most likely phenomenon?
Correct Answer : A
This is thought withdrawal. Which is the delusional belief that thoughts have been 'taken out' of his/her mind. It is commonly associated with thought block. But the difference is that in the thought block, there is a sudden break of a chain of thought and no one is stealing the ideas.
Q.86. A 30 year old schizophrenic female attacks her mother believing that aliens have replaced her with an exact double. What is the most likely condition she is suffering from?
Correct Answer : A
Capgras syndrome : A type of delusional misidentification in which the patient believes that a person known to them has been replaced by a ‘double’ who is to all external appearances identical, but is not the ‘real person’.
Q.87. A 33 year old man with a history of severe depression says his insides are rotting and nobody has bothered to bury him. He feels the world no longer exists and nothing matters. His gaze is always downwards and has barely any eye contact with anyone. What term describes his condition?
Correct Answer : A
Explanation given in the previous questions
Q.88. A 26 year old woman is afraid to visit the shopping centre. Crowds and public places cause her panic. She even feels distress, short of breath and develops palpitations when she boards the train to travel to see her aunt. She feels more relaxed when she goes out to public places with her husband than when alone. Her anxiety has been worsening over the past few months and she now refuses to go out becaue of this irrational fear. What is the most likely diagnosis?
Correct Answer : C
The answer here is agoraphobia as she has a fear of going out into the open.
Agoraphobia : Fear of open spaces
Q.89. A 33 year old woman has been feeling low and having difficulty in concentrating since her husband passed away 6 weeks ago. She has been crying almost everyday, and feeling hopeless. She has been withdrawing from other people and does not want to go out for dinner with her mother. What is the most likely diagnosis?
Correct Answer : A
An adjustment disorder occurs when an individual is unable to adjust to or cope with a particular stress or a major life event.
Q.90. A 35 year old male is bitterly annoyed with everyone around him. He complains that they are putting ideas into his head. What is the most likely phenomenon?
Correct Answer : B
Thought insertion is the delusional belief that thoughts are being placed in the patient’s head from outside. It is a first-rank symptom of schizophrenia.
Q.91. A 29 year old woman ias overly paranoid that her partner is being unfaithful to her. She checks his phones, email accounts and bank statements several timesa day for evidence of infidelity. She dislikes him going out as she fears that he would look at other woman while he is out on his own. She does not allow any social media for fear that he may meet other women. What is the most likely diagnosis?
Correct Answer : D
Please see the previous questions
Q.92. A 45 year old woman presents with complaints of abdominal pain and blood in the stool. She brings the stool sample from home but has never been able to produce a sample at the hospital. A urinalysis was done which was negative. Her blood tests are normal. This is the third time she is presenting to the hospital in the last month. On examination, multiple scars on the abdomen consistent with laparoscopies are seen. She insists on getting further investigations although no abnormalities are found. What is the most likely diagnosis?
Correct Answer : C
Munchausen's syndrome would fit best. Likely she is adding blood into her stools at home for medical attention. This can easily be done with a pin prick on the finger.
Munchausen's syndrome is also known as factitious disorder. Patients intentionally falsify their symptoms and past history and fabricate signs of physical or mental disorders with the primary aim of obtaining medical attention and treatment.
The diagnostic features are the intentional and conscious production of signs, falsification, or exaggeration of the history, and the lack of gain beyond medical attention and treatment.
Q.93. A 36 year old woman contacts the police to notify them that she was responsible for a recent disastrous flood which had resulted in loss of lives. What is the most likely kind of delusions she is suffering from?
Correct Answer : C
Delusion of guilt involves feeling guilty or remorseful for no valid reason. An example would be someone who believes they were responsible for a war in another country or hurricane damage in another state. The object of delusion believes that they deserve to be punished for their sins.
Q.94. A 22 year old woman comes to clinic with an obsessive ideation that she is overweight. Her body mass index is 17 kg/m2. She is on a strict diet and does aerobic exercises 3 times a day. While taking a history, she admits to laxative abuse. On examination, she has patches of thickened calluses on the dorsal aspect of the first interphalangeal joint. What is the most appropriate action?
Correct Answer : C
The thickened calluses on the knuckles are from self-induced vomiting which is called “Russell’s sign. It may be found on examination in patients with bulimia nervosa or anorexia nervosa. She falls into the category of moderate anorexia. Admission to the hospital is not warranted because her blood pressure and heart rate are fine and her BMI is still above 15. Referral to an eating disorder unit or service would be the most appropriate.
Q.95. A 64 year old man has a firm belief that the person on the billboard outside his window is sending messages that are meant specifically for him. What is the most likely type of delusion that this man is suffering from?
Correct Answer : C
Explanation discussed in previous questions.
Q.96. A 33 year old woman has been feeling down for the past one year. She feels fatigue and is eating more than usual. Several times a week she would wake up during the night and would not be able to go back to sleep. Occasionally, she hears voices of her late husband who died two years ago. What is the most likely diagnosis?
Correct Answer : A
This affected individual has lost her husband. Depression should be something to consider. The auditory hallucinations are signs that this woman is having some form of psychosis. The most probable diagnosis here is psychotic depression. Very commonly in exam, they would give a scenario of a person having symptoms of an atypical or typical depression plus having some form of hallucinations.
In the majority of the cases, the answer to that scenario would be psychotic depression.
Q.97. A 37 year old woman believes that her neighbours have been using her shower while she is away at work. She is convinced that they dry the bathroom and escape just before she goes into the bathroom. Her husband comes to share the same belief and informs the police. What is the most appropriate term for these symptoms?
Correct Answer : D
Folie à deux is the best term to describe this. It is a symptom of a delusional belief and hallucinations that are transmitted from one individual to another usually by two people in a close relationship like in this case, husband and wife. Folie à deux is French for “madness of two”.
It is a situation where two people with a close relationship share a delusional belief.
This arises as a result of a psychotic illness in one individual with the development of a delusional belief, which comes to be shared by the second. The delusion resolves in the second person on separation, the first should be assessed and treated in the usual way.
Q.98. A 64 year old woman has been brought by her sone for psychiatric evaluation. She says that she has stopped living with her husband because she is convinced that it is someone else posing to be him. What is the most likely condition she is suffering from?
Correct Answer : D
Given above
Q.99. A 55 year old man is brought to the GP surgery by his wife for a review of a growth on his forehead. His wife wants the growth removed but he refuses and says that the growth helps him think clearly. What is the most appropriate next course of action?
Correct Answer : A
Assessment of his mental capacity to refuse treatment would be the most appropriate next course of action. If his state of mind is fine, we can follow his instructions and leave the lesion there. But obviously, his mental state is not fine as he believes that this growth helps him think clearly. Mini Mental Status Examination (MMSE) is to assess patients with dementia.
All patients with cognitive impairment should be assessed with a Mini-Mental Status Examination (MMSE) to identify the areas of cognitive impairment.
Q.100. A 56 year old chronic alcoholic man wants to attend his daughter’s wedding in 2 weeks and does not want to be drinking during the wedding. He says he is determined to quit drinking alcohol but he wants extra help. What is the most appropriate medication?
Correct Answer : A
Acamprosate, in combination with counseling, may be helpful for maintaining abstinence in alcohol-dependent patients. It is useful for patients who are concerned that strong cravings will result in relapse. Clinical psychologists and mental health support groups are the wrong places to refer a chronic alcoholic.
Q.101. A 32 year old female presents to her GP with feelings of low mood. It was determined during the consult that she didn’t want to attend the appointment but came because her husband insisted. She describes a 4 month period of feeling low. In addition, she describes experiencing difficulty falling asleep at night and feeling lethargic all the time. She mentions not having much of an appetite anymore. She used to spend lots of her spare time writing children’s book but she says that she does not find pleasure in that anymore. She works as a financial consultant and says that her concentration at work has become impaired and that her work is suffering as a result. The GP made a diagnosis of moderate depression and started her on a four week course of fluoxetine. After four weeks, the patient returns with the same symptoms of low mood and no improvement in functional ability. What is the next best step in management for this patient?
Correct Answer : B
If a patient with diagnosed depression has been taking first-line antidepressant therapy (SSRIs) at a therapeutic dose for at least two to four weeks and presents with no improvement in symptoms after that period, there are steps that the clinician must take to manage the patient.
Q.102. A 68 year old man comes into A&E confused and with nausea, confusion and a coarse tremor. As the emergency doctor is taking a history, he slowly becomes unconscious. His wife gives a history that he took an overdose of a medication but she is unsure which medication it was. Which of the following medications could account for his symptoms?
Correct Answer : D
Lithium is a mood stabilising drug used in bipolar disorder. Its features include a coarse tremor (a fine tremor is seen in therapeutic doses), acute confusion and coma if severe.
Q.103. A 33 year old man tries not to go outside his house because he thinks that people will look at him and talk about him. He finds difficulty when talking with his peers in a restaurant or under social settings. He sweats and feels palpitations when speaking to strangers. He dislikes going to malls as he fears strangers may criticise his looks. He avoids these situations as they cause him distress. What is the most likely diagnosis?
Correct Answer : C
Sufferers of social phobias (social anxiety disorders) can have anxiety when going out of the house and when seen in public. When doing so, they fear of doing something embarrassing like looking incompetent, stupid, or unentertaining.
The distractor here is agoraphobia. Agoraphobia is the fear of open spaces which can also be seen when sufferers go to the shopping mall. However, the trigger for this fear is more of the inability to escape the open space rather than the fear of being criticized.
Q.104. A 38 year olf woman who gave birth 6 weeks ago presents to her local GP surgery with her husband. She describes ‘crying all the time’ and ‘not bonding’ with her baby. She is worried about baby’s health constantly and she is unsure if is able to cope with this new change in her life. What is the most likely diagnosis?
Correct Answer : C
Postnatal depression is seen commonly among women affecting up to 10% of women giving birth. Crying, not bonding, worrying, and feeling as if she cannot cope a typical presentations of postnatal depression.
Q.105. A 17 year old girl who was ‘fine’ until her boyfriend ended their relationship. Out of anger, she took 10 tablets of paracetamol after drinking alcohol. She is brought into A&E by her mother. What is the most appropriate next course of action?
Correct Answer : A
Referral to a psychiatric team would be the most accurate option here. Acute alcohol consumption is an inhibitor of the P-450 enzyme system. Since she has consumed alcohol acutely, the risk of fatal effects of paracetamol poisoning would be reduced.
Not to mention that 10 tablets of paracetamol in a 17-year-old is not life-threatening. She does however need a psychiatric evaluation before she leaves the hospital as she was clearly trying to harm herself.
Q.106. A 36 year old woman was recently admitted to a psychiatric ward. She believes that the staff and other patients know exactly what she is thinking all the time. What is the most likely phenomenon?
Correct Answer : A
Thought broadcasting is the delusional belief that one’s thoughts are accessible directly to others. It is found in schizophrenia
Q.107. A 37 year old woman who delivered 3 days ago is now concerned about her mood. She has trouble sleeping and feels generally anxious and tearful. She is unable to explain why she is crying al the time. She has no history of mental health disorders in the past. What is the most appropriate management?
Correct Answer : D
The diagnosis here is baby-blues which is seen in around 3/4 of women after delivering. Whilst poor sleep could potentially be a sign of depression, poor sleep is expected with a new baby.
Q.108. A 22 year old man is distressed that he hears voice of his deceased uncle telling him that he is being spied on. He feels low in mood and anxious. He has not left the house for 2 weeks and has recently started to drink increasing quantities of alcohol due to his anxiety. His speeches are interrupted with silence for a few seconds followed by topics unrelated to what was being discussed. He feels he is no longer in control of his own body and thoughts. What is the most suitable medication to treat his symptoms?
Correct Answer : C
This individual is suffering from schizophrenia. The speeches that are interrupted with silence followed by an unrelated topic are called thought blocking. The feature that he is no longer in control of his own body and thoughts is called the passivity phenomenon.
Both of these are features of schizophrenia. First-line treatment in newly diagnosed schizophrenia involves the use of newer atypical antipsychotics like olanzapine or risperidone.
Benzodiazepines are only used if rapid tranquillization is needed. This may be the choice if the scenario involves a violent, aggressive patient.
Q.109. A 24 year old man finds it difficult to come out of a room without having to turn the light switch off and on 3 times. He has tried more than several times to go out of the room without having to do this particular compulsion however he still returns to the room feeling agitated that it was not done. What is the most appropriate management?
Correct Answer : A
The diagnosis here is obsessive-compulsive disorder (OCD). Exposure and response prevention (ERP) is included in cognitive behavioural therapy (CBT) in treatment for those who present with OCD.
Q.110. A 52 year old woman speaks rapidly without any pause and ignores interruptions. She barely even pauses to take enough breaths. What is the best term to describe this kind of person?
Correct Answer : B
Pressure of speech The pressure of speech is a tendency to speak rapidly and frenziedly as if motivated by an urgency not apparent to the listener. The speech is rapid and difficult to interrupt, and, with the increasing severity of illness, the connection between sequential ideas may become increasingly hard to follow. Pressure of speech is a hallmark of mania and is often seen during manic periods in patients with bipolar disorder.
Q.111. A 33 year old man with alternating mood swings and episodes of mood elevation to depression underwent treatment and improvement was seen in his mood swings. What medication is needed to be continued to prevent his alternating moods?
Correct Answer : B
Please see Q -3
Q.112. A 24 year old man feels down and lethargic. In the last couple of months, he has stopped enjoying his hobbies which include playing the violin. He was admitted to the psychiatry ward last year following an episode of overspending, reckless behaviour and promiscuity. What is the most likely diagnosis?
Correct Answer : C
Please see Q-3
Q.113. A 57 year old man had a myocardial infarction 6 months ago. He has been having low moods since then and a diagnosis of moderate depression has been established. Which is the most appropriate medication to start him on?
Correct Answer : A
For the majority of patients with moderate depression, selective serotonin reuptake inhibitors (SSRIs) are considered first-line If this question gave you options for SSRIs, Sertraline would be the best answer to pick as sertraline has a good safety profile with patients with myocardial infarction.
The other antidepressants have not been studied enough in the context of myocardial infarction however citalopram has gained popularity and is also considered safe for use in patients with depression with a history of myocardial infarction. Citalopram and fluoxetine are the preferred SSRIs if there is no relevant past medical history.
Q.114. A 32 year old schizophrenic lady complains that she hears voices saying “she is evil”. What is the most likely type of hallucinations involved?
Correct Answer : C
Third-person auditory hallucinations Auditory hallucinations are characteristic of schizophrenia where voices are heard referring to the patient as ‘he’ or ‘she’, rather than ‘you’. For example "She is an evil person". Sometimes the voices are of a running commentary.
For example, the patient hears one or more voices providing a narrative of their current actions, ‘he’s getting up…now he’s going towards the window’.
Q.115. A 28 year old woman comes in with her limbs paralysed after witnessing a car accident. She cannot recall what just happened. What is the most likely diagnosis?
Correct Answer : D
Conversion (dissociative) disorders typically involve loss or disturbance of normal motor or sensory function which initially appears to have a neurological or other physical cause but is later attributed to a psychological cause.
The patient does not consciously feign the symptoms or seek material gain. Memory loss, seizures, loss of speech, and paralysis can occur.
Q.116. A 24 year old man finds it difficult to come out of his house without checking if he has locked the door several times. When he exits his house, he has to check it at least 3 times by pushing the door handle to confirm it is locked. He now wants help as he has been doing this for a couple of years. What is the most appropriate management?
Correct Answer : A
The diagnosis here is obsessive-compulsive disorder (OCD).
Exposure and response prevention (ERP) is included in cognitive behavioural therapy (CBT) in treatment for those who present with OCD. The method is predicated on the idea that a therapeutic effect is achieved as subjects confront their fears and discontinue their escape response.
Q.117. A 21 year old man is seen in the Psychiatry clinic with the complaint of strange body movements. His mother describes his body movements as unconventional and inappropriate. They consist of lip-smacking, protrusion of his tongue and quick, shaking movements of his hands and feet. The patient cannot control these movements. His past medical history is significant for schizophrenia, diagnosed a year ago. The patient states that he was started on medication for his schizophrenia several months ag however he does not take his medication regularly as he sometimes forgets to take it. What is the best treatment for this patient’s condition?
Correct Answer : B
The patient has tardive dyskinesia likely caused by his treatment with antipsychotic drugs. A common side effect of long-term use of antipsychotic drugs is tardive dyskinesia which is a movement disorder characterised by continuous involuntary movement of the tongue and lower face. Patients are often unaware of these movements and they are reported by family members like in this case, his mother.
Q.118. A 27 year old woman finds herself with palpitations and dizziness whenever she is in a meeting at the office. She is very self-conscious and feels that her colleagues are judging her in a harsh way in meetings. She has been asked to present in one of the meetings but she called in sick to avoid being criticised. What is the most likely diagnosis?
Correct Answer : D
The phrase to pick up in this question stems is the fear of “being criticized”. This is a good example of social phobia (social anxiety disorder). Sufferers avoid situations like meetings, group events, and public speaking as they may bring about the possibility of embarrassment.
Q.119. A 28 year old female who delivered 6 weeks ago feels sad and has no interest to feeding the baby. She has been eating poorly and having difficulty sleeping. She feels weak throughout the day and has stopped taking the baby out of the house. She also says that the baby has evil eyes. What is the most likely diagnosis?
Correct Answer : C
She has features of depression: feels sad, not eating well, has difficulty sleeping, feeling weak. On top of that she has delusional ideas: no interest to feed the baby, she thinks the baby has evil eyes and not taking the baby out of the house. These points to postpartum psychosis.
A key feature to look out for in EXAM is the words “evil eyes”. If a mother has recently delivered and thinks that her baby has evil eyes, pick the answer that has postpartum psychosis. You are most likely to be correct.
Postpartum psychosis usually starts with postpartum depression. Classically, they would have thoughts of harming their newborn baby. She would have delusional ideas that the baby is deformed, evil, or otherwise affected in some way and she has intent to kill or harm the baby. Onset usually within the first 2-3 weeks following birth.
Management :
• Admission to the hospital (specialist mother-baby unit if possible) is usually required
• For major affective disorder there is good evidence for ECT.
Q.120. A 35 year old is agitated and euphoric. He claims to be helping the prime minister with economic policies and describes himself to be a very powerful man. He believes that he has made important discoveries regarding international policies that have great impact towards the United Kingdom. On further investigation, these statements are untrue. What is the most likely phenomenon?
Correct Answer : D
His delusions describe a type of delusion called grandiose delusions which are usually found in patients with mania.
Q.121. A 42 year old man with a history of bipolar disorder is noted to have high serum levels of lithium and profound hypokalaemia on routine examination. He was recently diagnosed with essential hypertension and his GP had started him on an antihypertensive medication. What is the most likely cause of the recent findings?
Correct Answer : A
The key here is to understand what medication can cause both hypokalaemia and lithium levels to rise. Thiazides (e.g. bendroflumethiazide) can cause hypokalaemia. Thiazide and related diuretics can cause a rapid rise in serum lithium levels leading to toxicity. Guidelines recommend avoiding concurrent use unless lithium levels can be closely monitored and the lithium dose adjusted as necessary.
Q.122. A 28 year old woman witnessed a tragic road traffic accident where a young boy was killed by a lorry. On the night of the event, she was not able to sleep as she had horrid thoughts of the event. The following morning, she woke up and was not able to see. She has no previous history of medical or psychological issues. What is the most likely diagnosis?
Correct Answer : A
Conversion (dissociative) disorders typically involve loss or disturbance of normal motor or sensory function which initially appears to have a neurological or other physical cause but is later attributed to a psychological cause. The patient does not consciously feign the symptoms or seek material gain. Memory loss, seizures, loss of speech, and paralysis can occur. In this case, she lost her vision.
Remember that patients with conversion disorder are not feigning the signs and symptoms. Despite the lack of an organic diagnosis. The person’s distress is very real and the physical symptoms the person is experiencing cannot be controlled at will. Fortunately, a good prognosis can be expected and she will likely recover from her visual symptoms.
Q.123. A 30 year old woman who suffered from depression a few years ago has recently spent a substantial amount of money buying new clothes. She goes out almost every night with her friends. She would not allow any of her friends to choose the restaurant for dinner as she believes she knows the best places to eat. She sleeps less than usual and fills her days with as many activities as she can. What is the most likely diagnosis?
Correct Answer : C
There is a history of depression and symptoms of mania. This would be a classic scenario where they would like you to pick Bipolar disorder.
Q.124. A 37 year old man was recently sent to jail for breaking all the windows of a shop with his bat. When the manager tried to stop him, he hit the manager on the head. He has a history of many convictions and has been imprisoned many times. He finds it difficult to keep close relationships. He has 2 boys with his ex-wife but does not contact them. What is the most likely diagnosis?
Correct Answer : D
Antisocial Personality disorder : Characterized by continuous antisocial or criminal acts, inability to conform to social rules, impulsivity, disregard for the rights of others, aggressiveness, and lack of remorse. They will typically be manipulative, deceitful, and reckless.
Like other types of personality disorder, antisocial personality disorder is on a spectrum, which means it can range in severity from occasional bad behavior to repeatedly breaking the law and committing serious crimes. Psychopaths are considered to have a severe form of antisocial personality disorder.
Q.125. An 18 year old boy was recently sent to juvenile detention center after he set his father’s car on fire. He lacks remorse for setting the car on fire and says he would do it again if he had the chance to. He has always found it difficult to conform to social rules and has no regard for the rights of others. What is the most likely diagnosis?
Correct Answer : A
Please see Q-124
Q.126. A 32 year old lady has recently become more active over the past year. She sleeps less and recently bought a house and 2 new cars. She notices that her sex drive has increased. She often starts a task but not able to finish it as she has difficulty in focusing on one task alone. What is the most likely diagnosis?
Correct Answer : B
The symptoms here are representative of hypomania which can be thought of as a milder form of mania. With hypomania, there are higher than normal energy levels but they do not generally lead to hospitalization whereas manic episodes usually last for a week or more and may result in hospitalization.
Usually, if the stem includes grandiose ideas, hallucinations, or delusions PLUS elevated moods, then pick mania, otherwise pick hypomania. It is especially important to remember that hypomania does NOT present with hallucinations or delusions.
This would often be the difference between picking mania and picking hypomania as the answer.
Q.127. A 22 year old female was brought to A&E by her friend with the complaint of nausea and flushing. They were at a birthday party and the patient had been drinking copious amounts of alcohol. The patient says that she can see spots of colour around her peripheral vision. A visual field examination and fundoscopy was performed and was determined to be normal. She has no other medical problems and takes paracetamol on occasion for headaches. During the consult, the patient complains of being excessively thirsty and she consumes approximately one litre of water. Her vitals are as follows: Blood pressure: 138/89 mmHg Heart rate: 122/min Respiratory rate: 22 breaths/min Temperature: 37.9 C. What is the most likely drug that this patient has taken?
Correct Answer : A
We are all very automated into picking LSD as the answer as soon as we see some form colour vision distortion in the stem but what happens when LSD is not part of the options. This patient has likely taken Ecstasy. She has no past medical problems so diphenhydramine (an antihistamine) and amiodarone are incorrect.
Q.128. A 29 year old woman diagnosed with schizophrenia is complaining that the children playing outside her garden car hear her thoughts. She says they know exactly what she is thinking at all times of the day. What is the most likely phenomenon?
Correct Answer : D
Thought broadcasting is the delusional belief that one’s thoughts are accessible directly to others. It is found in schizophrenia.
Q.129. A 35 year old schizophrenic man hears voices narrating his actions like “he is going to the toilet” and “he is leaving the house”. What is the most likely type of hallucinations involved?
Correct Answer : C
Please see Q - 114
Q.130. A 22 year old woman was brought to the emergency department by her boyfriend with her fist bleeding after punching a mirror. She is distressed because he wants to end the relationship. Scars of old cuts on her forearms was noticed during a physical examination. She denies trying to end her life. What is the most likely diagnosis?
Correct Answer : B
Please see Q-32
Q.131. A 21 year old man was brought by his friends unconscious from a party where he was said to have drunk vodka. While he was being attended to by the doctor in the Emergency Department, he became conscious and said the green tie the attending doctor was wearing was talking to him. His friends also mention that an hour earlier he mentioned that he was seeing sounds coming out of the radio. On examination, his pupils are dilated. What substance could this patient have taken which could account for his symptoms?
Correct Answer : D
Hallucinations like this are common with patients on Lysergic acid diethylamide (LSD). Whenever you see a stem with odd sensations such as smelling colours or seeing sound, think LSD.
Q.132. A 20 year old woman with amenorrhoea and a body mass index of 14.8 kg/m2 is still try to lose weight. She exercises excessively and induces vomiting after her meals. She feels fatigued most of the time and has little interest in social events. What is the most likely diagnosis?
Correct Answer : A
The above described is anorexia nervosa. A low BMI and the refusal to maintain a normal body weight for her age are key towards the diagnosis. Fatigue and loss of interest in social events are also features of anorexia nervosa.
Q.133. A 60 year old man was brought to A&E by his daughter following ingestion of a large amount of medications. His daughter says that he has been withdrawn lately and that he has been neglecting himself. The patient himself says that he is tired of life and mentions that he has not left his house in the past three weeks as he claims that his neighbours are talking about him. He also claims that his neighbours have evil thougths toward him. The patient concedes that he needs help. What is the most appropriate next step in management of this patient?
Correct Answer : B
The most important thing to glean from this question is that this patient has conceded that he has a problem and that he needs help. This single fact would be grounds for a voluntary hospital admission. He needs a general hospital admission, voluntary or otherwise, for the very fact that he has ingested a large quantity of medications.
The type of medication was not given in the stem but it would be dangerous to send him elsewhere.
Q.134. A 33 year old woman has persistent fear when she has to speak publicly. She sweats and has palpitations and finds it very difficult to breathe. She is afraid of what people might think of her. She tries her best to avoid these situations. What is the most likely diagnosis?
Correct Answer : C
One of the hallmarks of social anxiety disorder is the intense fear of public speaking. They have a fear of being criticized. Sufferers fear that they may look stupid or appear boring.
Q.135. A 33 year old man who lives with his mother, always thinks when traffic lights turn red, his mother is calling him to come home. This is followed by his actions to drive back home. What is the most likely diagnosis?
Correct Answer : C
He thinks that the changing of traffic lights is giving him a message. These are called delusional perceptions and are a feature of schizophrenia.
Q.136. A 24 year old male on remand in prison for murder is referred by the prison doctor. He is noted to be behaving oddly in prison and complains of hallucinating. He has a previous history of IV drug abuse. On questioning, he provides repeated wrong answers to questions nonetheless, his answers are in the correct category. An example, when asked who is the prime minister of England, he answers Bill Clinton. What is the most likely diagnosis?
Correct Answer : D
Ganser syndrome is a type of factitious disorder, a mental illness in which a person deliberately and consciously acts as if he or she has a physical or mental illness. They mimic behavior that is typical of a mental illness, such as schizophrenia. It is also sometimes called prison psychosis because the syndrome occurs most frequently in prison inmates, where it may represent an attempt to gain leniency from prison or court officials.
Q.137. A 28 year old schizophrenic man refuses to let his father into the house because he has the delusion that his father has been replaced by an identical looking imposter. He easily recognised other family members but would misidentify his father. What is the most likely condition he is suffering from?
Correct Answer : A
Capgras syndrome - A type of delusional misidentification in which the patient believes that a person known to them has been replaced by a ‘double’ who is to all external appearances identical, but is not the ‘real person’.
Q.138. A 74 year old man is depressed after his wife’s death 6 months ago. He has been neglecting himself and is not eating well. He has lost 11 kg in the last 3 months. At times, he has thought about self harm but has never done it. His son found him in a very miserable state when he went to visit him last night. Unfortunately, the son is unable tocare for his father due to work and other family related issues. What is the most appropriate management?
Correct Answer : D
The first question to ask yourself is, is this a suitable environment for a person with a depressive illness. Obviously not, and thus he should be admitted to the psychiatry ward. Admission should be to a ward where close observation and monitoring are possible, whenever there is a significant risk of harm to self (or others) The ward environment is often not the quiet sanctuary patients hope for and this may lead to difficult decisions in balancing the risk of self-harm against the use of compulsory admission.
Q.139. A 29 year old teacher was run down by a drunk driver a year ago. Since then, she has been afraid to cross the road. She suffers from nightmares about that incident and gets a startled response every time she hears loud sounds. What is the most appropriate initial management?
Correct Answer : A
Please see Q-16
Q.140. A 62 year old man has uncontrollable lip smacking, puckering, repetitive chewing movements and tongue thrusting. He also has uncontrollable slow writhing movements of his fingers. He is on long-term antipsychotic medications for the past 15 years as part of the management of his schizophrenia. What is the most likely diagnosis?
Correct Answer : C
Tardive dyskinesia occurs especially in patients with long-term treatments with antipsychotic agents. The word “tardive” means delayed, and “dyskinesia” means abnormal movement. Put together “tardive dyskinesia” refers to the delay between the first use of offending medication and the onset of abnormal movements.
It takes several months to years to develop the symptoms of tardive dyskinesia from the use of antipsychotics. It is characterized by continuous involuntary movement of the tongue and lower face. The disorder can also affect the fingers, arms, and legs causing either rapid, jerking movements or slow, writhing movements.
Huntington’s disease usually presents at an earlier age with tremors and choreiform movements.
Parkinsonism can also be caused by antipsychotic medications however it is characterized by resting tremors, rigidity, and bradykinesia. The onset of parkinsonism as an extrapyramidal side effect of antipsychotics usually occurs after 1 week of administration of the offending antipsychotic drug.
Akathesia like tardive dyskinesia and Parkinson’s can be caused by the use of antipsychotic medications but the symptoms involve motor restlessness. These are restlessness of the lower limbs where the patient would have the urge to move. The word “akathisia” is derived from Greek and it means “not to sit still”.
Q.141. A 41 year old male patient was rushed to Accident & Emergency (A&E) with a Glasgow Coma Scale (GCS) of 13/15 following an incident that happened during a long haul flight. The patient was flying from Thailand to the United Kingdom when he started feeling nauseous followed by having a tremor and becoming confused. He has been taking regular ibuprofen 400 mg for his lower back pain for the past week. The patient’s past medical history is significant for bipolar disorder. He has been taking lithium for the past 10 years and is well controlled on it. What is the most appropriate test to do for this patient?
Correct Answer : A
The single best test to do for this patient would be to confirm his suspicion of lithium toxicity with a serum lithium level. Lithium is mostly excreted by the kidneys, and drugs such as NSAIDs increase renal reabsorption of lithium thus increasing serum lithium levels. Chronic accumulation of lithium such as in this case would usually result in mild to moderate symptoms.
Although a full battery of tests such as a full blood count, typically blood glucose and renal function tests would be done in real life, if asked for “the most APPROPRIATE test” the examiners would like you to consider lithium toxicity as a diagnosis, and request a serum lithium concentration.
Q.142. A 25 year old female has just been diagnosed with bipolar disorder. The psychiatric team is considering commencing her on lithium carbonate treatment. Before being commenced on lithium, the patient is requied to go for a variety of blood tests. A blood request form has been given to her for calcium levels and a renal function thest. What is the most appropriate test to send this patient for in addition to the ones listed?
Correct Answer : D
Before treatment with lithium is commenced, tests are needed to assess both kidney function and thyroid function. The kidneys mostly excrete lithium with minimal metabolism by the liver. In addition, it can also cause long-term kidney damage, so kidney monitoring is done at regular intervals.
A thyroid function test is done initially because abnormal levels of thyroid hormones can mimic or precipitate mania and depression. It is also good practice to have a baseline prior to starting lithium.
Thyroid monitoring is done at regular intervals because lithium can induce hypothyroidism. Following the initial test, anyone on lithium should have their lithium levels checked 3 monthly, and TSH and renal function checked at least 6 monthly if not more frequently.
Q.143. A 45-year-old man keeps having intrusive thoughts about having dirt under the bed. He can’t keep himself from thinking about these thoughts. If he tries to resist, he starts having palpitations. What is the most likely diagnosis?
Correct Answer : B
Obsessive-Compulsive Disorder (OCD) is a disorder in which a person has uncontrollable, reoccurring thoughts (obsessions) and behaviors (compulsions) that he or she feels the urge to repeat over and over. Here patient’s thoughts are obsession and though no compulsive act is described (like repeated cleansing of dirt) but his nature of thought like inability to resist thinking or getting palpitations on trying to avoid thinking can be regarded as compulsion of thought.
CBT is 1st line treatment and antidepressants (SSRIs) are tried if no response to CBT.
More severe OCD may require a longer course of therapy and/or medication.
Q.144. 37-year-old lady strongly believes that a famous politician has been sending her flowers every day and is in love with her. However, this is not the case. What is the most likely diagnosis?
Correct Answer : A
Erotomania is a type of delusion in which the affected person believes that another person, usually a stranger, high-status or famous person, is in love with them]. Pyromania is an impulse control disorder in which individuals repeatedly fail to resist impulses to deliberately start fires, in order to relieve tension or for instant gratification.
Kleptomania is the inability to refrain from the urge to steal items. Trichotillomania is an impulse disorder characterized by the compulsive urge to pull out one's hair, leading to noticeable hair loss and balding. Grandiosity refers to an unrealistic sense of superiority.
Q.145. A 35-year-old man with a history of schizophrenia is brought to the ER by his friends due to drowsiness. On examination he is generally rigid. A diagnosis of neuroleptic malignant syndrome can be made excepting which of the following?
Correct Answer : D
Usually occurs after prolonged treatment. [Neuroleptic malignant syndrome is a life-threatening reaction that can occur in response to neuroleptic or antipsychotic medication.
Symptoms include high fever, confusion, rigid muscles, variable blood pressure, sweating, and fast heart rate. Neuroleptic malignant syndrome usually occurs SOON after the start or in increasing doses of antipsychotic drugs.
All other conditions mentioned are seen in this syndrome like renal failure, pyrexia, elevated creatinine kinase, tachycardia, etc.
Q.146. A 35-year-old male is bitterly annoyed with people around him. He thinks that people are putting ideas into his head. What is the most likely diagnosis?
Correct Answer : B
Thought insertion is feeling as if one's thoughts are not one's own, but rather belong to someone else and have been inserted into one's mind. It is a common feature of schizophrenia.
Q.147. 35-year-old man who has served in the army presents with lack of interest in enjoyable activities and feeling low. He doesn’t feel like reading the news or watching movies as he believes there is violence everywhere. What is the most appropriate first line therapy?
Correct Answer : C
This is mild depression. In mild depressive illness CBT is the preferred treatment.
Q.148. A 32-year-old woman in tears describing constant irritability with her 2 small children and 261 inability to relax. She describes herself as easily startled with poor sleep and disturbed nightmares following a house fire a year ago, while the family slept. What is the best treatment?
Correct Answer : D
The diagnosis is Post Traumatic Stress Disorder. Guidelines suggest that trauma-focused psychological therapies (CBT or EMDR) should be offered before medication, wherever possible.
As these are not in option the best answer here is Fluoxetine (SSRI).
At present, there is evidence that EMDR, psychotherapy, behaviour therapy, and antidepressants are all effective.
There is not enough information for us to say that one of these treatments is better than another.
Q.149. A patient with thought disorder washes hands 6 times each time he uses the toilet. What is the best management?
Correct Answer : B
The diagnosis is obsessive-compulsive disorder. For obsessive-compulsive disorder CBT is the 1st treatment of choice and if fails next comes drugs.
Q.150. A 78-year-old man is depressed after his wife’s death. He has been neglecting himself. His son found him in a miserable state when he went to visit. The son can’t deal with his father. What is the appropriate management?
Correct Answer : A
Voluntary admission to the psychiatry ward.
Q.151. A 29-year-old teacher is involved in a tragic RTA. After that incident, he has been suffering from nightmares and avoided driving on the motorway. He has been diagnosed with PTSD (post traumatic stress disorder). What is the most appropriate management?
Correct Answer : A
CBT is the treatment of choice in PTSD. Guidelines recommends that you are offered 8–12 regular sessions of around 60–90 minutes, seeing the same therapist at least once a week.
Q.152. A 23-year-old girl presented with perioral paresthesia and carpopedal spasm 20 minutes after a huge argument with her boyfriend. What is the next step for this patient?
Correct Answer : C
Hyperventilation causes CO2 washout and respiratory alkalosis. An increase in pH, alkalosis, promotes increased protein binding, which decreases free calcium levels and this low calcium causes perioral paresthesia and carpopedal spasm.
If you continue breathing and rebreathing in the paper bag it will allow CO2 concentration to rise in the paper bag and as you rebreath this again and again you will regain some washed-out CO2 and thus relief to this alkalosis.
Q.153. A 25-year-old woman has been feeling anxious and nervous for the last few months. She also complains of palpitations and tremors. Her symptoms last for a few minutes and are very hard to control. She tells you that taking alcohol initially helped her relieve her symptoms but now this effect is wearing off and she has her symptoms even after drinking alcohol. What is the diagnosis?
Correct Answer : A
Panic disorder is an anxiety disorder characterized by reoccurring unexpected panic attacks. Panic attacks are sudden periods of intense fear that may include palpitations, sweating, shaking, shortness of breath, numbness, or a feeling that something terrible is going to happen.
Q.154. A 7-year-old child is very naughty. His teacher complains that he is easily distracted. His parents say that he can’t do a particular task for a long time. He sometimes hurts himself and breaks many things. This causes many troubles at home. What is the diagnosis?
Correct Answer : C
Attention deficit hyperactivity disorder (ADHD) is a condition that affects people's behaviour. People with ADHD can seem restless, may have trouble concentrating, and may act on impulse.
Symptoms of ADHD tends to be noticed at an early age and may become more noticeable when a child's circumstances change, such as when they start school. Most cases are diagnosed when children are 6 to 12 years old.
The symptoms improve with age, but many adults continue to experience problems.
Q.155. A 47-year-old ex-soldier suffers from low mood and anxiety. He can’t forget the images he faces before and has always had flashbacks. He is not able to watch the news because there are usually some reports about war. What is he suffering from?
Correct Answer : B
Repeated flashbacks and tendency to avoid the thoughts of stressors is diagnostic of PTSD.
PTSD is diagnosed after a person experiences symptoms for at least one month following a traumatic event. However, symptoms may not appear until several months or even years later. The disorder is characterized by three main types of symptoms: Re-experiencing the trauma through intrusive distressing recollections of the event, flashbacks, and nightmares.
Emotional numbness and avoidance of places, people, and activities that are reminders of the trauma. Increased arousal such as difficulty sleeping and concentrating, feeling jumpy, and being easily irritated and angered.
Q.156. A 36-year-old woman has recently spent a lot of money on buying clothes. She goes out almost every night with her friends. She believes that she knows better than her friends, so she should choose the restaurant for eating out. She gave history of having low mood at 12 years ago. What’s the diagnosis? .
Correct Answer : C
Initial depressive episode (may be befor a long) followed by mania is bipolar affective disorder.
Q.157. A 25-year-old woman was brought to the emergency department by her boyfriend. She has many superficial lacerations on her forearm. She is so distressed and constantly says her boyfriend is going to end the relationship. She denies trying to end her life. What is the most likely diagnosis?
Correct Answer : D
Borderline personality disorder: Act impulsively and develop an intense but short-lived emotional attachment to others. They are usually attention seekers but not suicidal.
Q.158. A 28-year-old business executive presents at the GP asking for some help because she has been arguing with her boyfriend frequently. She is worried about her weight, and she thinks she may be fat. She has been on a diet and lost 7 kgs in the last 2 months on purpose. She is eating less. She used to do a lot of exercise. Now she says she’s feeling down, has some insomnia and feels tired and without energy. She has not showed up at work. She is worried because recently she got a loan to buy a luxury car. She can’t be fired. She complains about her low mood. She thinks this is weird because she used to be extremely productive. She used to work showing an excellent performance at the office. She even received compliments from her boss. Now, she says her boyfriend is angry because her apartment is a chaos. Usually she spends a lot of time cleaning it, even upto 3 AM. She liked it to be perfect, but not it’s a mess. On examination: BMI 23, no other signs. What is the most probably diagnosis?
Correct Answer : B
Now she is depressed but before hypomanic which makes the likely diagnosis of bipolar disorder.
Q.159. A woman brought her husband saying she wants the ‘thing’ on his forehead removed. The husband is refusing treatment saying it improves his thinking. What is the most appropriate next step?
Correct Answer : A
Assess his mental capacity to refuse treatment.
Q.160. A 37-year-old man who has many convictions and has been imprisoned many times has a history of many unsuccessful relationships. He has 2 boys but doesn’t contact them. What is the most probable diagnosis?
Correct Answer : D
Antisocial personality disorder is a particularly challenging type of personality disorder, characterized by impulsive, irresponsible, and often criminal behavior.
Q.161. A young female who has many superficial lacerations was brought into the emergency department by her boyfriend for superficially lashing her upper arm. She is adamant and screaming that she is not suicidal but scared her boyfriend wants to leave her. What is the diagnosis?
Correct Answer : D
Borderline personality disorder (BPD), also known as emotionally unstable personality disorder, is a long-term pattern of abnormal behavior characterized by unstable relationships with other people, unstable sense of self, and unstable emotions. There is often an extreme fear of abandonment, frequent dangerous behavior, a feeling of emptiness, and self-harm.
Q.162. A 22-year-old woman was brought by her boyfriend with multiple superficial lacerations. There are scars of old cuts on her forearms. She is distressed because he wants to end the relationship. She denies suicide. What is the most likely diagnosis?
Correct Answer : B
Borderline personality disorder (BPD), also known as emotionally unstable personality disorder is a long-term pattern of abnormal behavior characterized by unstable relationships with other people, unstable sense of self, and unstable emotions.
There is often an extreme fear of abandonment, frequent dangerous behavior, a feeling of emptiness, and self-harm.
Q.163. A 31-year-old single man lives with his mother. He usually drives to work. He always thinks when the traffic lights change, his mother is calling him, so he drives back home. What is the diagnosis?
Correct Answer : C
The deliusion metioned is a delusion of perception - he thinks that the changing traffic lights are giving a message to him and this is often seen in schizophrenic patients (one of the 1st rank symptoms).
Q.164. A nurse comes to you saying that she has recently developed the habit of washing her hands after every 15-20 minutes. She is unable to concentrate on her work and takes longer than before to finish tasks as she must constantly wash her hands. What is the most appropriate management?
Correct Answer : A
The likely diagnosis is obsessive-compulsive disorder. Most effective treatment for obsessive-compulsive disorder is cognitive behavioral therapy.
Q.165. A 28-yer-old business man came to the sexual clinic. He was worried that he has HIV infection. 3 HIV tests were done and all the results are negative. After a few months, he comes back again and claims that he has HIV. What is the diagnosis?
Correct Answer : B
Hypochondria is a condition in which a person is excessively and unduly worried about having a serious illness.
Q.166. A 19-year-old man has been happier and more positive than usual, with more energy than he has ever felt before for no particular reason. He has been getting more work done at the office today and has been socializing with his friends as usual. What is the most likely diagnosis?
Correct Answer : D
Features like i) elevated mood ii) more energy than before iii) getting more work done at the office (loss of inhibition) is common for both mania and hypomania. This is not mania as in mania we shall get psychotic symptoms like :
i) delusion of grandeur and
ii) auditory hallucinations, which are absent here.
Q.167. A 35-year-old female attempts suicide 10 times. There is no history of psychiatric problems and all neurological examinations are normal. What is the best treatment?
Correct Answer : A
The patient is not psychotic and with normal neurology! So she may be having some problem in family life, finance, job, social life or somewhere like this which she is not able to cope with and that is leading to her suicidal thoughts.
Q.168. A 34-year-old woman presents 3 weeks after childbirth. She has had very low mood and has been suffering from lack of sleep. She also has thought of harming her little baby. What is the most appropriate management for this patient?
Correct Answer : B
Diagnosis is postpartum depression. Intrusive thought of harming the baby is a feature of postpartum depression. CBT should be tried first.
Antidepressants if CBT does not work or the patient does not want CBT. If severe depression and all other fail ECT is considered.
Q.169. A 36-year-old man walks into a bank and demands money claiming he owns the bank. On being denied, he goes to the police station to report this. What kind of delusions is he suffering from?
Correct Answer : D
A delusion of grandeur is the false belief in one's own superiority, greatness, or intelligence. People experiencing delusions of grandeur do not just have high self-esteem; instead, they believe in their own greatness and importance even in the face of overwhelming evidence to the contrary.
Q.170. An old man comes to the doctor complaining that a part of this body is rotten and he wants it removed. What is the most likely diagnosis?
Correct Answer : D
Nihilistic delusion: parts of the body do not exist or are dead.
Guilt: an emotion that occurs when a person feels that they have violated a moral standard.
Hypochondriasis: worry about having a serious illness.
Munchausen’s: a psychiatric factitious disorder wherein those affected feign disease, illness, or psychological trauma to draw attention, sympathy, or reassurance to themselves.
Capgras syndrome: a delusion that a friend, spouse, parent, or other close family member (or pet) has been replaced by an identical-looking impostor.
Q.171. A woman presents with complains of abdominal pain, unsteadiness, numbness of lower limb and palpitations. All investigations are normal. What is the diagnosis?
Correct Answer : B
This is multiple, recurrent, medically unexplained symptoms usually starting early in life. Usually patient presents with one symptom at a time. Investigations are normal.
Q.172. A 28-year-old female who delivered 6 weeks ago feels sad and has no interest to feeding the baby. She has been eating poorly and having difficulty sleeping. She feels weak throughout the day and has stopped taking the baby out of the house. She also says that the baby has evil eyes. What is the most likely diagnosis?
Correct Answer : C
Features of depression: feels sad, poor eating, difficulty sleeping, feeling weak ii) delusional ideas: thinks baby has evil eyes and not taking the baby out of the house. These features point to the diagnosis of postpartum psychosis.
Postpartum psychosis starts within 2 weeks (occasionally later) of delivery and it can take 6 -12 months or more to recover from postpartum psychosis.
Q.173. A man is very depressed and miserable after his wife’s death. He sees no point in living now that his wife is not around and apologises for his existence. He refuses any help offered. His son has brought him to the emergency department. The son can’t deal with the father any more. What is the most appropriate next step?
Correct Answer : B
This patient is refusing any help offered! And his son cannot deal with him anymore! In this situation voluntary admission to the psychiatry ward is not possible and the option of choice is “compulsory admission under MHA”.
The point here is the man has felt himself in danger by self-neglect.
Q.174. A 19-year-old boy is brought by his mother with complaint of lack of interest and no social interactions. He has no friends, he doesn’t talk much, his only interest is in collecting cars/vehicles having around 2000 toy cars. What is the most appropriate diagnosis?
Correct Answer : D
Autistic spectrum disorder. Autism spectrum disorders affect three different areas of a child's life:
1. Social interaction
2. Communication - both verbal and nonverbal
3. Behaviors and interests In some children, a loss of language is a major impairment.
In others, unusual behaviors (like spending hours lining up toys) seem to be the dominant factors.
Q.175. A man with a family history of panic disorder is brought to the hospital with palpitations, tremors,sweating and muscles tightness on 3 occasions in the last 6 weeks. He doesn’t complain of headache and his BP is within normal limit. What is the most appropriate long-term treatment for him?
Correct Answer : D
Recommended treatment for panic disorder is i) CBT ii) Medication (SSRIs or TCA). NICE recommends a total of seven to 14 hours of CBT to be completed within a four month period. Treatment will usually involve having a weekly one to two-hour session.
When the drug is prescribed usually an SSRI is preferred. Antidepressants can take two to four weeks before becoming effective.
Q.176. A 56-year-old woman who is depressed after her husband died of cancer 3 months ago was given amitryptaline. Her sleep has improved and she now wants to stop medication but she still speaks about her husband. How would you manage her?
Correct Answer : B
Depression is an important feature of bereavement. Patients may pass sleepless nights.
As this patient's sleep has improved it indicates she has a good response to antidepressants and as she still speaks about her husband there is a chance of deterioration of her depression if an antidepressant is stopped. For depressive episodes antidepressants should be continued for at least 6-9 months.
Q.177. A schizophrenic patient hears people only when he is about to fall asleep. What is the most likely diagnosis?
Correct Answer : B
Hypnagogic or hypnopompic hallucinations are visual, tactile, auditory, or other sensory events, usually brief but occasionally prolonged, that occur at the transition from wakefulness to sleep (hypnagogic) or from sleep to wakefulness (hypnopompic).
Q.178. A 32-year-old man has obsessive compulsive disorder (OCD). What is the best treatment?
Correct Answer : A
CBT
Q.179. A 65-year-old woman says she died 3 months ago and is very distressed that nobody has buried her. When she is outdoors, she hears people say that she is evil and needs to be punished. What is the most likely explanation for her symptoms?
Correct Answer : C
Psychotic depression, also known as depressive psychosis, is a major depressive episode that is accompanied by psychotic symptoms (hallucinations, delusions).
In this patient, nihilistic delusion favours the diagnosis of psychotic depression. It can occur in the context of bipolar disorder or major depressive disorder.
Q.180. A 40-year-old woman has had intermittent tension, dizziness and anxiety for 4 months. Each episode usually resolves after a few hours. She said she takes alcohol to make her calm. She is in a loving relationship and has no problems at work or home. What is the next step in her management?
Correct Answer : A
Collateral information
Q.181. A 71-year-old woman looks disheveled, unkempt and sad with poor eye contact. She has recently lost her husband. Which of the following describes her condition?
Correct Answer : D
Low mood
Q.182. A 23-year-old man comes to the emergency department with a history of drug misuse. He recognizes that he has a problem and is willing to see a psychiatrist. Which of the following terms best describes this situation?
Correct Answer : D
In psychiatry, the patient's awareness and understanding of the origins and meaning of his attitudes, feelings, and behavior and his disturbing symptoms (self-understanding) is known as insight.
Q.183. A 24-year-old man asks his GP for a sick note from work. He says that he feels down, is lethargic and has stopped enjoying playing the piccolo (his main hobby). He was admitted to the psychiatry ward last year following an episode of overspending, promiscuity and distractibility. What is the most probable diagnosis?
Correct Answer : C
Presently patient has depression and previous features of mania make the likely diagnosis of bipolar affective disorder.
Q.184. A 40-year-old woman presents to the GP with low mood. Of note, she has an increased appetite and has gone up 2 dress sizes. She also complains that she can’t get out of bed until the afternoon. What is the most likely diagnosis?
Correct Answer : D
Atypical depression is a subtype of major depression or dysthymic disorder that involves several specific symptoms, including increased appetite or weight gain, sleepiness or excessive sleep, marked fatigue or weakness, moods that are strongly reactive to environmental circumstances, and feeling extremely sensitive to rejection.
Q.185. A 26-year-old woman had bipolar disorder for 10 years and is on Lithium for it. She is symptom free for the past 4 years. She is now planning her pregnancy and wants to know whether she should continue taking lithium. What is the most appropriate advice?
Correct Answer : D
Reduce lithium gradually and stop when pregnancy is confirmed.
Q.186. A 22-year-old man has rushed into the emergency department asking for help. He describes recurrent episodes of fearfulness, palpitations, faintness, hyperventilation, dryness of the mouth with peri-oral tingling and cramping of the hands. His symptoms last 5-10 minutes and have worsened since their onset 3months ago. He is worried he may be having a heart attack. An ECG shows sinus tachycardia. What is the most appropriate immediate intervention?
Correct Answer : C
Patient has an anxiety disorder (panic) which causes hyperventilation and CO2 washout leading to respiratory alkalosis.
Symptoms will improve by rebreathing into a paper bag as it will cause a gradual increase of CO2 in the paper bag and decrease the severity of respiratory alkalosis.
Q.187. A 21-year-old woman has had several sudden onset episodes of palpitations, sweating, nausea and overwhelming fear. On one occasion she was woken from sleep and feared she was going insane. There is no previous psychiatric disorder. What is the most probable diagnosis?
Correct Answer : B
Here closest d/d to panic disorder is pheochromocytoma. But in pheochromocytoma, the most important feature is resistant hypertension and other important features are headache and abdominal pain which all are absent here. Moreover overwhelming fear is more suggestive of panic disorder. Hence here the diagnosis is Panic disorder.
Q.188. A thin 18-year-old girl has bilateral parotid swelling with thickened calluses on the dorsum of her hand. What is the most likely diagnosis?
Correct Answer : A
18 year thin girl, bilateral parotid swelling and thickened calluses on the knuckles from self induced vomiting are suggestive of Bulimia nervosa. Bulimia often is associated with bilateral parotid swelling (parotid hypertrophy).
Q.189. A 32-year-old lady complains that she hears everyone saying that she is an evil person. What type of hallucinations is she suffering from?
Correct Answer : B
Third-person hallucinations are auditory hallucinations in which patients hear voices talking about themselves, referring to them in the third person, for example "he is an evil person".
Q.190. A 36-year-old woman was recently admitted to a psychiatric ward. She believes that the staff and other patients know exactly what she is thinking all the time. What is the most likely symptom this patient is suffering from?
Correct Answer : D
Thought broadcasting
Q.191. A 60-year-old woman is admitted to the hospital after a fall. She is noted to have poor eye contact. When asked how she is feeling, she admits to feeling low in mood and losing enjoyment in all her usual hobbies. She has also found it difficult to concentrate, feels that she is not good at anything, feels guilty over minor issues and feels very negative about the future. What is the most likely diagnosis?
Correct Answer : A
Mild depression: i)Low mood ii) Anhedonia iii) Guilt iv) Hopelessness v) Worthlessness vi) Inability to concentrate.
Q.192. A 19-year-old man presents for the 1st time with a firm and unshakable belief that he is being followed by terrorists who are plotting against him. What is the best term for this man’s condition?
Correct Answer : A
Delusions of persecution refer to false beliefs or perceptions in which a person believes that they are being treated with malicious intent, hostility, or harassment – despite significant evidence to suggest otherwise.
Q.193. A 19-year-old female is brought in by her parents. They are concerned about her BMI which is 12. She is satisfied with it. What is the next step?
Correct Answer : D
The diagnosis is anorexia nervosa. At this critical low BMI medical admission is indicated to improve her deficiency states and proper nutrition. BMI <15kg/m2, rapid weight loss + evidence of system failure requires urgent referral to eating disorder unit (EDU), medical unit (MU) or paediatric medical wards.
Q.194. A 52-year-old woman speaks rapidly without any pause and ignores interruptions. She doesn’t even pause to take enough breaths. What term best describes this kind of speech?
Correct Answer : D
Pressure of speech is a tendency to speak rapidly and frenziedly as if motivated by an urgency not apparent to the listener. The speech produced is difficult to interrupt. Such speech may be too fast, erratic, irrelevant, or too tangential for the listener to understand. Commonly seen in hypomania, mania, schizophrenia, extreme anxiety , and attention deficit hyperactivity disorder.
Q.195. A 30-year-old woman has been feeling low and having difficulty in concentrating since her mother passed away 2 months ago. She feels lethargic and tends to have breathlessness and tremors from time to time. What is the most likely diagnosis?
Correct Answer : A
When 2 months passed it is no more normal bereavement but major depression or adjustment disorder.
Q.196. A 50-year-old man presents with low mood, poor concentration, anhedonia and insomnia. He has had 2 episodes of increased activity, promiscuity and aggressive behavior in the past. He was arrested 8 months ago for trying to rob a bank claiming it as his own. Which drug is most likely to benefit him?
Correct Answer : D
It is a case of bipolar disorder. The mainstay of treatment is mood stabilizers such as i) Lithium carbonate ii) Anticonvulsant medicines iii) Antipsychotic medicines. So from the given options, Carbamazepine is the most appropriate drug.
Q.197. A 55-year-old woman who attends the clinic has recently been diagnosed with a depressive episode. She complains of unintentionally waking early in the morning, a recent disinterest in sex and a loss of appetite, losing 5kg weight in the last month. She 281 feels that her mood is worse at the beginning of the day. What is the most likely diagnosis for this patient?
Correct Answer : B
Sleep disturbance, disinterest in sex and loss of appetite points towards the diagnosis of moderate depression.
Q.198. A 31-year-old woman presents with 7-10 days following childbirth, with loss of feeling for the child, loss of appetite, sleep disturbance and intrusive and unpleasant thoughts of harming the baby. What is the best treatment for this patient?
Correct Answer : A
"Intrusive thought" of harming the baby is postpartum depression and attempt to harm is postpartum psychosis. So the given case is postpartum depression and the answer is A. Fluoxetine.
Q.199. A 24-year-old woman presents with episodes of peri-oral tingling and carpo pedal spasms every time she has to give a public talk. This also happens to her before interviews, exams and after arguments. What is the best management strategy for this patient?
Correct Answer : C
Desensitization is the treatment of choice in the long run. For prevention of propranolol before expected exposure and if a patient presents with an attack then rebreathing in a paper bag to subside her acute problems.
Q.200. A schizophrenic says: life is unfair. I like fairs. Fairs have food. It must be good. What term describes this patient’s speech?
Correct Answer : D
The rhymic use of words as described is known as clanging or clang association often seen in schizophrenia.
Q.201. A patient suffering from schizophrenia laughs while talking about his father’s death. Which term best describes his condition?
Correct Answer : D
Incongruent affect means inappropriate emotional response like laughing hearing sad news or crying hearing good news.
Q.202. A 24-year-old woman is afraid to leave her house as whenever she goes out, she tends to have SOB (shortness of breath) and sweating. She has stopped going out except with her husband. What is the most likely diagnosis?
Correct Answer : D
Agoraphobia: Extreme or irrational fear of open or public places.
Q.203. A woman is sad, fatigues and she is eating more and also has sleeping disturbance and hears the voice of her husband who died 3years ago. What is the diagnosis?
Correct Answer : B
Here features of atypical depression along with hallucination make the likely diagnosis to be psychotic depression.
Q.204. A woman with a history of drug abuse and increased alcohol intake, now comes for help and she is concerned about her problem. What is the most appropriate management option?
Correct Answer : B
Psychiatry team
Q.205. A 25-year-old woman with a history of several episodes of depression is brought to the emergency department after she was found with several empty bottles of her medications. She complains of coarse tremor, nausea and vomiting. Which of the following drugs is likely to have caused her symptoms?
Correct Answer : C
Side effect of lithium is fine tremor (in therapeutic dose) but lithium toxicity or lithium poisoning causes coarse tremor. Coarse tremors, nausea, and vomiting are well-known features of lithium overdose.
Q.206. A 23-year-old man feels anxious and agitated when faced with stress. He has an interview in 3 days and would like some help in relieving his symptoms. What is the most appropriate management?
Correct Answer : C
Inappropriate anxiety during interview is performance phobia which is better helped by propranolol when help is needed for short term like here as 3 days.
Q.207. A woman presents with a history of poisoning 10 times with different substances. There are no obvious signs of depression or suicidal behavior. What is the best preventive step?
Correct Answer : D
Patient is not depressed and there is no suicidal behaviour. Despite repeated poisoning may indicate she is facing some stress so insight into her problem should be sought for.
Q.208. A 24-year-old male on remand in prison for murder is referred by the prison doctor. He is noted to be behaving oddly whilst in prison and complains of seeing things. He has a previous history of IV drug abuse. On questioning he provides inappropriate but England. What is the prisoner suffering from?
Correct Answer : C
Ganser syndrome is characterized by nonsensical or wrong answers to questions or doing things incorrectly often with visual pseudohallucinations and a decreased state of consciousness. It is also sometimes called prison psychosis, because the syndrome occurs most frequently in prison inmates, where it may represent an attempt to gain leniency from prison or court officials.
Q.209. A 32-year-old lady has recently become more active, sleep less and bought a house and 2 new cars. What is the most likely diagnosis?
Correct Answer : C
Individuals in a hypomanic state have a decreased need for sleep, are extremely outgoing and competitive, have a great deal of energy and are otherwise often fully functioning (unlike full mania where unlike fully functioning there is a. obvious difficulties at work or in social relationships and activities b. requires admission to hospital to protect the person or others, or c. the person is suffering psychosis).
Q.210. A 20-year-old girl with amenorrhea and BMI of 14 still thinks she has to lose weight. What is the most likely diagnosis?
Correct Answer : A
Anorexia nervosa, often referred to simply as anorexia, is an eating disorder, characterized by low weight, food restriction, fear of gaining weight, and a strong desire to be thin. Many people with anorexia see themselves as overweight even though they are, in fact, underweight.
Q.211. A guy who has several convictions and has been imprisoned several times, breaks up with his family and doesn’t contact his children. What type of personality disorder is this?
Correct Answer : B
Antisocial personality disorder is characterized by a pervasive pattern of disregard for, or violation of, the rights of others. There may be an impoverished moral sense or conscience and a history of crime, legal problems, and impulsive and aggressive behavior.
Q.212. A 49-year-old man lost his job and now is homeless. He was found wandering in the park. He is muttering that some people are after him. Alcohol was tested and it was negative. What will your next step be?
Correct Answer : B
As the patient has persecutory delusion he is likely suffering from psychosis for which neuropsychoanalysis is the logical approach.
Q.213. A 24-year-old woman known to be suffering from panic disorder presents to the hospital with tingling and numbness in her fingers. ABG: pH 7.52, PCO2 2.2kPa, PO2 11kPa, Bicarb 20. What is the most likely condition?
Correct Answer : C
In panic attack there occurs hyperventilation which causes a washout of CO2 and leads to acute respiratory alkalosis resulting in raised pH >7.45 (here 7.52), low PCO2 (here 2.2 kPa) with compensatory decrease in HCO3 (here 20meq/l).
Q.214. A 48-year-old woman always socially withdrawn has stopped going out of the house. She is afraid to socialize because she fears that people will criticize her. What is the most probable diagnosis?
Correct Answer : C
Social anxiety disorder is a type of complex phobia. This type of phobia has a disruptive or disabling impact on a person's life. It can severely affect a person's confidence and self-esteem, interfere with relationships and impair performance at work or school.
Q.215. A 43-year-old woman presents with low mood, loss of libido, sleep disturbance, tiredness, palpitation, chest discomfort, irritability and recurrent worries. What is the most likely diagnosis?
Correct Answer : B
Mild depression: i)Low mood ii) Anhedonia iii) Guilt iv) Hopelessness v) Worthlessness vi) Inability to concentrate.
Treatment is CBT (cognitive behavioral therapy)
Moderate depression:
Features of mild + vii) Poor sleep viii) Poor Appetite ix) Poor libido x) Easy fatiguability.
Treatment is Antidepressants.
Q.216. A young man has been found in the park, drunk and brought to the emergency department by ambulance. He recently lost his job and got divorced. He thinks nurses are plotting against him. What is the most likely diagnosis?
Correct Answer : D
Psychotic depression
Q.217. A 26-year-old political refugee has sought asylum and complains of poor concentration. He keeps getting thoughts of his family whom he saw killed in a political coup. He is unable to sleep and feels hopeless about his survival. Because of this he is afraid to go out. What is the most likely diagnosis?
Correct Answer : B
Keep getting thoughts of his family disaster, and insomnia, to avoid similar circumstances he is avoiding going out these are suggestive of PTSD.
Q.218. A 32-year-old man with schizophrenia and a history of violence and distressing auditory hallucinations was admitted to the ward with aggressive behavior and has already smashed his room. He is refusing any oral meds. What is the most appropriate injection?
Correct Answer : C
If oral preparation was asked we could give risperidone but in question asked for injectable preparation. Injection Risperidone is a depot preparation the action of which takes a few days to start. So in acute cases, it is not suitable; hence from the given options haloperidol is the best choice.
Q.219. A 24-year-old woman has severe depression 3 months after the birth of her first child. She is breastfeeding but is otherwise unable to look after the baby and is convinced that her family is likely to kill her. She has no interest in anything and keeps crying. What is the most appropriate treatment?
Correct Answer : D
Diagnosis is given in question as severe depression and as features of psychosis present the better diagnosis would be psychotic depression. As onset is beyond 4 weeks it is no more post partum psychosis. So the 1st line treatment for psychotic depression is combination of antipsychotic + antidepressant. As the combination is not given as options first choice goes for antipsychotic that is D. Haloperidol.
Q.220. A 75-year-old man has been attending the clinic for lower urinary tract symptoms. His mood is very low and he says he feels unhappy, anxious and unable to sleep. He has been diagnosed with moderate depression. What treatment would be most effective for this patient?
Correct Answer : B
Moderate depression is treated with antidepressants. Amitryptiline and Dosulepine cause urinary retention (which is comparatively less in citalopram) so not suitable for a patient with lower urinary symptoms. Hence Citalopram is the preferred option.
Q.221. An 18-year-old girl has been diagnosed with anorexia nervosa and has mild depressive symptoms. She has cut down her food intake for the last 18 months and exercises 2 hours everyday. Her BMI 15.5, BP 90/60mmHg. What would be the most appropriate management?
Correct Answer : A
Anorexia nervosa is moderate when BMI is 15- 17.5 as in given case. In moderate anorexia (BMI 15–17.5, no evidence of system failure) routine referral can be to the local community mental health team or Eating Disorder Unit (EDU) if available. So the key is A. Eating disorder clinic.
Q.222. A 37-year-old woman believes that her neighbours have been using her shower while she is away from home. Her 42 year old partner is convinced about this and calls the police. What term best describes this situation?
Correct Answer : D
Folie a deux is a symptom of a delusional belief and hallucinations that are transmitted from one individual to another as here from wife to husband.
Q.223. A 30-year-old woman has experienced restlessness, muscle tension and sleep disturbance on most days over the last 6 months. She worries excessively about a number of everyday events and activities and is unable to control these feelings which are impairing her ability to hold down her job. What is the most likely diagnosis?
Correct Answer : B
In GAD patient is worried about a different number of events every day. Almost everything triggers the anxiety.
Q.224. A 24-yer-old schizophrenic has been under antipsychotic treatment for the last 1 year and now complains of ED Erectile Dysfunction). Which drug is most likely to have caused this?
Correct Answer : D
Haloperidol raises serum prolactin levels to 20~40 ng/ml in therapeutic doses but risperidone causes around 171 ng/ml. High prolactin is responsible for erectile dysfunction.
Q.225. A 45-year-old woman has recently been diagnosed with MS (multiple sclerosis) and has been started on oral steroids. She is brought to the hosp after having ingested 100 paracetamol tablets 4h ago. She is refusing allmedical treatment. What is the next best step?
Correct Answer : B
Refer to psychiatrist to assess patients ability to refuse treatment.
Q.226. A 24-year-old lady has been low after the death of her husband and had stopped contacting her family. She was started on SSRI treatment and starts feeling better after a few months. On discontinuating themedicine she starts feeling that she has developed cancer just like her husband. What is the most appropriate next step?
Correct Answer : C
As the patient has developed new symptoms like fear of cancer, neuro-psychiatric analysis is needed.
Q.227. A person doesn’t go outside the home because he thinks that people will look at him and talk about him. He finds it difficult to associate with his peers in a restaurant or under social settings. What is the most likely diagnosis?
Correct Answer : D
Social phobia, is an anxiety disorder characterized by a significant amount of social anxiety (fear in one or more social situations), causing considerable distress and impaired ability to function in at least some parts of daily life.
Q.228. A 68-year-old woman has been admitted with poor appetite, weight loss, poor concentration and self neglect for 3 weeks. She has not been eating or drinking adequately and has rarely left her bed. She is expressive suicidal ideas and is convinced that people are out to kill her. She has been on antidepressant therapy for the past 3 months with no improvement. What is the most appropriate treatment?
Correct Answer : D
For psychotic depression try antidepressants and if needed augment by adding antipsychotics and if failed then ECT. But if there is suicidal ideation ECT should be given which is proven to be safe and effective.
Q.229. A 25-year-old girl saw a tragic RTA in which a young boy was killed. The night of the event she couldn’t sleep and the day after she suddenly lost her vision. She was previously fine and there was no history of medical or psychological problems. What is the diagnosis?
Correct Answer : A
Conversion reaction is sometimes applied to patients who present with neurological symptoms, such as numbness, blindness, paralysis, or fits, which are not consistent with a well-established organic cause, and which cause significant distress. It is thought that these symptoms arise in response to stressful situations affecting a patient's mental health.
Q.230. A 57-year-old man who had MI a few months ago has been having a low mood. A diagnosis of moderate depression has been established. Which medication is the best treatment for him?
Correct Answer : A
Among SSRIs Sertraline is the drug of choice in MI patients. If SSRI cannot be used Mirtazapine is recommended as the next antidepressant.
Q.231. A 36-year-old woman has recently spent a lot of money on buying clothes. She goes out almost every night with her friends. She believes that she knows better than her friends, so she should choose the restaurant for eating out with her friends. She gave history of having low mood at 12 years. What is the diagnosis?
Correct Answer : C
In bipolar disorder there is depressive disorder alternate with mania and the present case is in the manic stage of bipolar disorder.
Q.232. A 45-year-old woman presents with complains of abdominal pain and blood in stool. She brings the stool sample from home but has never been able to produce a sample at the hospital. Her urine and blood tests are normal. Examination: multiple scars on the abdomen consistent with laparoscopies and appendectomy. She insists on getting further inv although no abnormalities are found. What is the most likely diagnosis?
Correct Answer : D
Munchausen syndrome is a psychiatric factitious disorder wherein those affected feign disease, illness, or psychological trauma to draw attention, sympathy, or reassurance to themselves.
Q.233. A 36-year-old woman contacts the police to notify them she was responsible for a recent disastrous flood with loss of lives. What kind of delusions is she suffering from?
Correct Answer : C
Delusions of guilt or sin (self-accusation): This type of delusion involves feeling guilty or remorseful for no valid reason.
An example would be someone that believes they were responsible for a war in another country or hurricane damage in another state. In this case, the person believes that they deserve to be punished for their sins and place full blame on themselves.
Q.234. A 27-year-old man presents with symptoms characterized by alternating mood swings along with flight of ideas, elation, over activity and disinhibition, or low mood with lack of energy and social withdrawal. What is the most probable diagnosis?
Correct Answer : A
Flight of idea, elation, overactivity, and disinhibition are features of mania and low mood, lack of energy and social withdrawal are features of depression. Alternating mood swings with depression and mania are characteristic of Bipolar disorder.
Q.235. A 64-year-old man believes a female newscaster is communicating directly with him when she turns a page. What kind of delusions is he suffering from?
Correct Answer : D
Delusions of reference – A neutral event is believed to have a special and personal meaning. For example, a person with schizophrenia might believe a billboard or a celebrity is sending a message meant specifically for them.
Q.236. A couple attends their GP because of marital problems. The wife states that her husband is having affairs although she has no proof of this. The husband states that she even had him followed by a private detective and this is putting considerable strain on their marriage. What is the most likely diagnosis?
Correct Answer : D
Othello syndrome is a delusion of infidelity (cheating, adultery, or having an affair) of a spouse or partner.
Q.237. A 26-year-old man strongly believes that every elderly man he meets is his father. Although they look different, he is sure it is father wearing different disguises. What kind of delusions is this man suffering from?
Correct Answer : D
The Fregoli delusion, or the delusion of doubles, is a rare disorder in which a person holds a delusional belief that different people are in fact a single person.
Q.238. A 22-year-old man keeps having persistent and intrusive thoughts that he is a dirty thief. No matter what he tries these thoughts keep coming to him. Any attempt to avoid these thoughts leads to serious anxiety. What is the most likely diagnosis?
Correct Answer : B
Though here is an obsession of thoughts but no compulsion of activity but there is the compulsion of thoughts as well which makes the diagnosis OCD.
Q.239. A 30-year-old man is becoming concerned about the safety of his family. He has been checking the locks of the door every hour during the night. He becomes very anxious if his wife tries to stop him. What is the most likely diagnosis?
Correct Answer : D
Obsessive compulsive disorder (OCD) is a mental health condition where a person has obsessive thoughts and compulsive activity.An obsession is an unwanted and unpleasant thought, image or urge that repeatedly enters a person's mind, causing feelings of anxiety, disgust or unease. A compulsion is a repetitive behaviour or mental act that someone feels they need to carry out to try to temporarily relieve the unpleasant feelings brought on by the obsessive thought.
Q.240. A patient presents with a mask face. He also has gait problems. Which class of drug is causing this?
Correct Answer : B
Anti-psychotics by reducing dopamine can precipitate or induce parkinsonism.
Q.241. A 22-year-old girl unhappy about her weight with BMI=22. She likes to have her dinner in an expensive restaurant. She does excessive shopping. Serum K+ 3.3. What is the diagnosis?
Correct Answer : D
BMI 22, even though unhappy, hypokalemia, like to have dinner in an expensive restaurant (probable binge eating) suggests the diagnosis of bulimia.
Q.242. A 22-year-old girl had a fight with her boyfriend and then took 22 tablets of paracetamol. She was commenced on N-acetyl cysteine and she was medically fit to go home the following day. Which of the following does she require?
Correct Answer : C
Suicidal ideation is abnormal and if discharged there is chance of repeated suicidal attempts. So inpatient referral to psychiatrist is needed for this patient.
Q.243. A 36-year-old pregnant woman comes for evaluation with her husband. Her husband has been complaining of morning sickness, easy fatiguability and even intermittent abdominal pain. What is the husband suffering from?
Correct Answer : B
Cauvade syndrome, also called sympathetic pregnancy, is a proposed condition in which a partner experiences some of the same symptoms and behavior of an expectant mother.
Q.244. A woman complaining of diarrhea, abdominal pain and fatigue. All the tests are found to be normal. What is the cause?
Correct Answer : A
Somatization disorder is patients presenting with any physical symptom and frequent medical visits in spite of negative investigations.
Q.245. A patient has had 1 episode of depression and 2 episodes of mania over the last year and now presents with depression. He is on anti-depressants. What additional pharmacological treatment would now act as a prophylaxis for his condition?
Correct Answer : C
Antidepressants alone can lead to manic attacks and to prevent this a mood stabilizer is required.
Q.246. A schizophrenic man complains that he can hear voices talking about him and telling him to end his life by cutting his throat. He only hears them when he wakes up from sleep and not at other times. What type of hallucinations is he having?
Correct Answer : D
Hallucinations during going to sleep is hypnagogic and hallucinations during waking up from sleep are hypnapompic hallucinations.
Q.247. A 28-year-old woman complains of hearing strange voices in her bedroom as she is falling asleep in the night. She says there is no one in the room except her. On evaluation she has no other problems. What is she suffering from?
Correct Answer : C
Hallucinations during going to sleep is hypnagogic and hallucinations during waking up from sleep are hypnapompic hallucinations.
Q.248. A 32-year-old man on psychiatric medications presents with coarse tremors and diarrhea. What is the most likely alternative to the drug causing the problem?
Correct Answer : A
Valproate is causing the problem (valproate causes coarse tremor in therapeutic dose) and an alternate drug is Lithium. As the question asks alternate drug answer is A. Lithium.
Q.249. A 43-year-old man with a history of hospital admissions talk about various topics, moving from one loosely connected topic to another. What is the most likely diagnosis?
Correct Answer : B
Flight of idea is common in mania.
Q.250. A 38-year-old woman is in the emergency department following an over dose of her medicines. She doesn’t want medical treatment for the over dose. She says she wishes to be discharged. What is the appropriate management?
Correct Answer : D
There is every chance of repeat overdose. So she should be admitted under the Mental Health Act.
Q.251. A man says his insides are rotting and nobody has buried him. Which term best describes his condition?
Correct Answer : A
Nihilistic delusions are specific psychopathological entities characterized by the delusional belief of being dead, decomposed, or annihilated, having lost one's own internal organs or even not existing entirely as a human being.
Q.252. A 20-year-old man complains that all his movements are being watched. Sometimes he feels as though his actions are being controlled by his radio. At other times he is aware of voices describing what he is doing. What is the most probable diagnosis?
Correct Answer : D
Schizophrenia
Q.253. A 35-year-old is agitated and euphoric. He claims to be helping the prime minister with economic policy, although this is not true when checked. What is the most likely diagnosis?
Correct Answer : A
Agitated, euphoric, and delusion of grandiosity makes the likely diagnosis to be “Mania”. Agitation and euphoria are not features of delusion of grandiosity but mania.
Q.254. A 22-year-old man says that he can hear the voice of his deceased uncle telling him that he is being spied on. The patient is distressed by this becoming low in mood and anxious and has not left the house for 2 weeks. He is starting to drink increasing quantities of alcohol. He is noticed to have thought-block and passivity phenomena. What is the most suitable medicine to treat his symptom?
Correct Answer : D
Auditory hallucination, social withdrawal, and thought block are features of schizophrenia. So olanzapine is the drug to be prescribed from the given option.
Q.255. A 50 year old man presents with flight of ideas which are rambling and disinhibited. He is distractable, confused and overactive. What is the most likely diagnosis?
Correct Answer : B
Flight of ideas, disinhibition, distractibility, confusion, and overactivity are features of mania.
Q.256. An 18-year-old woman says that she can’t walk around as she is very big for that room. What is the most likely hallucination?
Correct Answer : C
Alice in Wonderland Syndrome (AIWS) is a paroxysmal, hallucinatory disorder with typical visual perceptual features. With fewer than 200 reported cases in the world, the syndrome is named after Lewis Carroll's famous book because, like Alice, patients with AIWS experience changes in the perception of their body dimensions, experiencing macro or microsomatognosia - the feeling of being bigger or smaller in relation to their environment.
Q.257. A 62-year-old man with chronic schizophrenia presents with a mask like face and involuntary pill rolling movement in both hands. He complains of chronic cough and forgetfulness. He is on long term antipsychotic meds. What is the most likely diagnosis?
Correct Answer : B
Antypsychotic drugs can cause parkinsonism.
Q.258. A 64-year-old woman has been brought by her son for psychiatric evaluation. She says that she has stopped living with her husband because she is convinced it is someone else posing to be him. What kind of delusions is she suffering from?
Correct Answer : D
The delusion of doubles, is a rare disorder in which a the person holds a delusional belief that different people are in fact a single person who changes appearance or is in disguise.
Q.259. A 19-year-old man accuses his friend of making his right arm swing out at a stranger. What is the best term to describe his condition?
Correct Answer : A
Delusion of control is a false belief that another person, group of people, or external force controls one's general thoughts, feelings, impulses, or behavior.
Q.260. A man on antipsychotic medicines develops features of retinitis pigmentosa. Which drug is most likely to cause these symptoms?
Correct Answer : A
Thioridazine and other antipsychotics (neuroleptics, dopaminergic antagonists) can cause degenerative retinopathies with histological, electrophysiological and symptomatological features similar to those of primary retinitis pigmentosa.
Q.261. A 39-year-old woman presents with symptoms recurring annually characterized by depressed mood, being socially withdrawn hypersomnia, lack of enjoyment in life, last for several months. What is the most likely diagnosis?
Correct Answer : A
Depression associated with late autumn and winter and thought to be caused by a lack of light.
Q.262. This condition affects middle aged women more than men and is characterized by low mood, early morning waking, loss of libido, tiredness and suicidal intention last for at least 2 weeks. What is the most probable diagnosis?
Correct Answer : C
The given case is severe depression (major depressive disorder).
Mild depression: i)Low mood ii) Anhedonia iii) Guilt iv) Hopelessness v) Worthlessness vi) Inability to concentrate. Treatment CBT.
Moderate depression: Features of mild + vii) Poor sleep viii) Poor Appetite ix) Poor libido x) Easy fatiguability. Treatment Antidepressant.
Severe depression: Features of moderate + xi) Suicidal intentions. Treatment ECT.
Psychotic depression: Features of severe + xii) Hallucinations xiii) Delusions xiv) Guilt xv) Nihilistic delusion. Treatment ECT.
Q.263. A 23-year-old single male was brought to Emergency exhausted and frightened. His father tells you that his son, who was previously healthy, had, for no apparent reason, a sudden attack of fear, dizziness, sweating, palpitations and the feeling that his heart is going to stop beating. The symptoms started to decrease gradually after about 10 minutes. Which is the most likely diagnosis?
Correct Answer : A
A sudden feeling of acute and disabling anxiety; often fear of death.
Q.264. A 60-year-old man is brought to the ED in an agitated state. He is lashing out violently. Which drug in low dosage due to its relative lack of autonomic side effects is a drug of choice in the treatment of agitation in this patient?
Correct Answer : B
Haloperidol has autonomic side effects.
Q.265. A 24-year-old man believes his bowels are blocked and his life is in ruin. What kind of delusion is he suffering from?
Correct Answer : D
The man's belief that his bowels are blocked and his life is ruined is an example of nihilistic delusion. A nihilistic delusion of the nonexistence or dissolution of a body part; in extreme form, the delusion of being dead or nonexistent.
Q.266. A man under psychiatric treatment develops GI distress and tremors. Which drug is most likely to cause these symptoms?
Correct Answer : A
Lithium causes tremors, GI distress (vomiting) along with Diabetes insipidus.
Q.267. A 30-year-old schizophrenic female attacks her mother believing that aliens have replaced her with an exact double. What condition is she suffering from?
Correct Answer : A
Capgras syndrome is an irrational belief that a familiar person or place has been replaced by a duplicate.
• Ganser syndrome is a fictitious disorder in which a patient deliberately acts as if he has a physical or mental illness when he doesn’t have it.
• Todd syndrome/Alice In Wonderland syndrome is a disorienting neurological condition affecting human perception of size, shape, and time.
• Fregoli syndrome is a delusion of doubles, a delusional belief that different ppl are infact a single person in disguise or change appearance.
Q.268. A patient comes with weight loss and sleep disturbance has mild depression. He has a history of MI. What is the most appropriate treatment?
Correct Answer : A
Treatment for mild depression is CBT. As the patient has MI imipramine is not suitable and in mild depression, an antidepressant is usually not given. As the patient has sleep disturbance diazepam can be given.
Q.269. A 45-year-old woman has been extensively investigated for a lump she believes to be cancer. She doesn’t think doctors take her seriously and demands another referral. What term best describes her condition?
Correct Answer : C
Worry about having a serious illness. This debilitating condition is the result of an inaccurate perception of the condition of body or mind despite the absence of an actual medical condition.
Q.270. A 40-year-old divorced man with bipolar affective disorder attends hospital following an OD (over dose) of 30 TCA (tricycle antidepressant) tablets. His new partner has left him and he has stopped taking his medicine and begun drinking heavily. He appears depressed, feels hopeless and is ambivalent about being alive. He is now fit for discharge from the medical ward and acknowledges the benefits of previous treatment. What is the most appropriate next management?
Correct Answer : A
To save the patient from another overdose.
Q.271. A 38-year-old man has disturbing thoughts about his house being infected by germs. He is anxious about safety and checks the locks of his doors repeatedly before going to bed. For the last 8 weeks he has been washing his hands every time he touches the lock, 20-30 times a day. What is the most appropriate management?
Correct Answer : D
This scenario describes a case of OCD (obsessive compulsive disorder) for which the best management is CBT followed by SSRIs or TCAs.
Q.272. A 35-year-old patient has been diagnosed with schizophrenia. He mimics the doctors and attendants – doing the same physical actions as them. What symptom does this patient have?
Correct Answer : A
Echopraxia is the involuntary repetition or imitation of another person's actions. Similar to echolalia, which is the involuntary repetition of sounds and language. Echopraxia has long been recognized as a core feature of Tourette syndrome and is considered a complex tic, but it also occurs in autism spectrum disorders, schizophrenia, and catatonia.
Q.273. A 65-year-old patient who had MI 1 year ago now comes to the emergency department complaining that his neighbor is conspiring against him. When his son is asked, he denies it and also narrates that sometimes his father says that everybody in his office is always talking about him, which is not the case. What is the most appropriate medication?
Correct Answer : C
Clozapine is associated with the highest risk of developing or aggravating metabolic syndrome, while olanzapine and risperidone carry intermediate risk of it. Metabolic syndrome is one of the crucial risk factors predisposing or aggravating patients to cardiovascular diseases.
Q.274. A young girl with a psychiatric history on medical treatment is brought to the dermatologist by her mother because of recurrent patchy hair loss. Exam: the hair shafts revealed twisting and fractures. This suggests the following pathology:
Correct Answer : D
Trichotillomania is a compulsive desire to pull out one's hair.
Q.275. A 68-year-old man presents with bruising and history of falls. He is found to have a mask-like face, pillrolling tremor and shuffling gait. EEG normal. Which of the following conditions is he most likely being treated for?
Correct Answer : C
Antipsychotics can lead to parkinsonism.
Q.276. A 45-year-old woman presents with easy fatigability, even on no exertion, chronic headaches and body aches and severe physical and mental exhaustion. She has no underlying conditions and all investigations are non-conclusive. What is the most likely diagnosis?
Correct Answer : B
Chronic fatigue syndrome (CFS) causes persistent fatigue (exhaustion) that affects everyday life and doesn't go away with sleep or rest.
Q.277. A 59-year-old woman whose periods stopped 5 years ago has become increasingly depressed. She now feels life is no longer worth living and threatens suicide. What is the most likely management plan?
Correct Answer : B
Patient is depressed with suicidal thoughts. She should be referred to psychiatrist.
Q.278. A 38-year-old woman has low mood, early morning awakening and poor appetite for six weeks. She had a similar episode 12 years ago and a further admission with over activity, irritability and grandiose. What is the most likely diagnosis?
Correct Answer : C
The patient has now features of depression and 12 years ago she had similar symptoms and later symptoms of mania. So she is suffering from bipolar affective disorder.
Q.279. A 40-year-old woman has acute symptoms of palpitations, sweating, difficulty in breathing and feeling unsteady on her feet. She has noticed her symptoms occur only in crowded public places. What is the diagnosis?
Correct Answer : C
One may have agoraphobia if he has fear of: Entering shops, crowds, and public places. Travelling in trains, buses, or planes. Being on a bridge. Being in a lift. Being in a cinema, restaurant, etc, where there is no easy exit. Being anywhere far from your home.
Q.280. A young woman fainted in the street and was brought the A&E. She weighs 33 kg. She is alert and well in the A&E and states she is on a diet and admits to not eating or drinking anything for 48 hours. She is annoyed that she has been brought to the hospital and adamantly states that she wants to leave. What is the most appropriate next step?
Correct Answer : A
Detain her under the Mental Health Act.
Choose a Question
×Choose a Question
×
Trial Access Limit Reached
You’ve reached the limit of free content. Subscribe to continue learning without restrictions.